Você está na página 1de 62

Cap tulo 13 Solu co es de Equa co es Diferenciais Ordin arias Lineares no Plano Complexo

Conte udo
13.1 Solu c oes em S eries de Pot encias para Equa c oes Regulares . . . . . . . . . 13.1.1 A Equa c ao do Oscilador Harm onico Simples . . . . . . . . . . . . . . . . . . 13.1.2 A Equa c ao de Legendre . . . . . . . . . . . . . . . . . . . . . . . . . . . . . . 13.1.3 A Equa c ao de Hermite . . . . . . . . . . . . . . . . . . . . . . . . . . . . . . 13.1.4 A Equa c ao de Airy . . . . . . . . . . . . . . . . . . . . . . . . . . . . . . . . 13.1.5 A Equa c ao de Tchebychev . . . . . . . . . . . . . . . . . . . . . . . . . . . . 13.1.6 O Caso de Equa c oes Regulares Gerais . . . . . . . . . . . . . . . . . . . . . . Solu c ao de Equa c oes Singulares Regulares. O M etodo de Frobenius . . . 13.2.1 Equa c oes Singulares Regulares. O Caso Geral . . . . . . . . . . . . . . . . . 13.2.2 A Equa c ao de Euler Revisitada . . . . . . . . . . . . . . . . . . . . . . . . . 13.2.3 A Equa c ao de Bessel . . . . . . . . . . . . . . . . . . . . . . . . . . . . . . . 13.2.4 Equa c oes Relacionadas ` a de Bessel. A Equa c ao de Bessel Esf erica . . . . . . 13.2.5 Equa c oes Relacionadas ` a de Bessel. A Equa c ao de Bessel Modicada . . . . 13.2.6 A Equa c ao de Laguerre . . . . . . . . . . . . . . . . . . . . . . . . . . . . . . 13.2.7 A Equa c ao Hipergeom etrica . . . . . . . . . . . . . . . . . . . . . . . . . . . 13.2.8 A Equa c ao Hipergeom etrica Conuente . . . . . . . . . . . . . . . . . . . . . Algumas Equa c oes Associadas . . . . . . . . . . . . . . . . . . . . . . . . . . 13.3.1 A Equa c ao de Legendre Associada . . . . . . . . . . . . . . . . . . . . . . . . 13.3.2 A Equa c ao de Laguerre Associada . . . . . . . . . . . . . . . . . . . . . . . . Exerc cios Adicionais . . . . . . . . . . . . . . . . . . . . . . . . . . . . . . . . APENDICES . . . . . . . . . . . . . . . Prova da Proposi c ao 13.1. Justicando os Polin omios de Legendre . . . . Polin omios de Legendre: Provando (13.14) . . . . . . . . . . . . . . . . . . Justicando os Polin omios de Hermite . . . . . . . . . . . . . . . . . . . . . Polin omios de Hermite: Provando (13.20) . . . . . . . . . . . . . . . . . . . Porque deve ser um Inteiro Positivo na Equa c ao de Laguerre . . . . . . Polin omios de Tchebychev: Obtendo (13.39) a Partir de (13.36)(13.38) . . . . . . . . . . . . . . . . . . . . . . . . . . . . . . . . . . . . . . . . . . . . . . . . . . . . . . . . . . . . . . . . . . . . . . . . . . . . . . . . . . . . . . . . . . . . . . . . . . . . . . . . . . . . . . . . . . . . . . . . . . . . . . . . . . . . . . . . . . . . . . . . . . . . . . . . . . . . . . . . . . . . . . . . . . . . . . . . . . . . . . . . . . . . . . . . . . . . . . . . . . . . . . . . . . . . . . . . . . . . . . . . . . . . . . . . . . . . . . . . . . . 569 570 571 574 576 578 581 583 . 586 . 593 . 595 . 605 . 607 . 608 . 610 . 614 616 . 616 . 618 620 621 621 622 624 625 626 628 . . . . . .

13.2

13.3

13.4 13.A 13.B 13.C 13.D 13.E 13.F

rataremos no presente cap tulo de apresentar solu co es de equa co es diferenciais ordin arias lineares e homog eneas, regulares ou com pontos singulares regulares. Por simplicidade, e para atender ao interesse de problemas f sicos, trataremos apenas de equa co es de segunda ordem mas, em ess encia, tudo o que faremos facilmente se generaliza para equa co es de ordem superior. Nossa abordagem estar a centrada no chamado m etodo de expans ao em s erie de pot encias (para equa co es regulares) e no m etodo de Frobenius (para equa co es com singularidades regulares). Estudaremos tanto casos gerais (com razo avel detalhe) quanto equa co es particulares de interesse em F sica. Em um certo sentido, o presente cap tulo d a continuidade ao Cap tulo 12, mas dele s o utilizaremos os Teoremas 12.3 e 12.4, das p aginas 531 e 535, respectivamente. Esses teoremas fundamentais s ao as justicativas dos m etodos de solu ca o que empregaremos. Comentamos ainda que trataremos as equa co es diferenciais como equa co es no plano complexo ainda que, na F sica, o interesse tipicamente resida em equa co es na reta real pois, como discutimos no Cap tulo 12, a natureza das solu co es e a justicativa dos m etodos de solu ca o s ao melhor entendidas quando abandonamos as limita co es da reta real de modo a explorar a estrutura anal tica das equa co es e suas solu co es.

568

JCABarata. Curso de F sica-Matem atica

Vers ao de 29 de janeiro de 2013.

Cap tulo 13

569/2069

Por vezes, omitiremos detalhes de c alculos e o estudante e convidado a complet a-los como exerc cio. Apesar de alguns desses c alculos omitidos serem reconhecidamente entediantes (n ao s o os omitidos, ali as), o estudante e recomendado faz elos ao menos uma vez durante sua exist encia terrena, pois n ao e poss vel apoderar-se do conhecimento aqui desenvolvido apenas por meio de leitura passiva. O tratamento que faremos de solu co es de equa co es gerais e bastante detalhado, um tanto mais do que o por vezes encontrado na literatura. Os resultados gerais est ao resumidos nos Teoremas 13.1 e 13.2, adiante. O tratamento de certas equa co es particulares de interesse em F sica (como as de Legendre, Hermite, Airy, Tchebychev, Bessel e Laguerre) e razoavelmente completo e v arias propriedades especiais das solu co es, tais como rela co es de ortogonalidade, rela co es de recorr encia, f ormulas do tipo de Rodrigues, representa co es integrais etc. (todas importantes na resolu ca o de problemas de F sica) s ao discutidas com detalhe no Cap tulo 14, p agina 630. Uma omiss ao e um estudo detalhado do comportamento assint otico de certas solu co es. Esperamos que futuramente essa lacuna possa ser completada. Exemplos selecionados de problemas de F sica onde algumas das equa co es particulares que discutimos se apresentam (e a conseq uente resolu ca o desses problemas) poder ao ser encontrados no Cap tulo 19, p agina 842, ao qual remetemos os estudantes interessados em adquirir um pouco de motiva ca o. A leitura daquele cap tulo requer um conhecimento parcial das solu co es das equa co es diferenciais e suas propriedades, de modo que o estudante dever a alternar sua leitura com a do material que a precede nos Cap tulos 13 e 14. Todas as equa co es particulares tratadas, suas solu co es e propriedades dessas solu co es, s ao amplamente discutidas na vasta literatura pertinente e a ela remetemos os estudantes interessados. Vide, por exemplo, [207], [257], [151], [9], [249], [45], [110], [111], [24], [50], [51], [70], [224], [107], [104]. Para uma abordagem da teoria das fun co es especiais sob o ponto de vista de teoria de grupos, vide [246].

13.1

Solu co es em S eries de Pot encias para Equa co es Regulares

Vamos na presente se ca o ilustrar o Teorema 12.3 da p agina 531 estudando a solu ca o por s erie de pot encias de algumas equa co es diferenciais ordin arias, homog eneas de segunda ordem e regulares de interesse (especialmente em F sica). Boa parte dos m etodos apresentados nos exemplos aplicam-se a equa co es de ordem maior que dois, mas n ao trataremos de tais generaliza co es aqui pois elas pouco apresentam de especial e seu interesse na F sica e reduzido. Na Se ca o 13.2, p agina 583, ilustraremos o Teorema 12.4, p agina 535, tratando de forma semelhante v arias equa co es singulares regulares de interesse pelo m etodo de Frobenius. Conforme demonstramos em p aginas anteriores (Teorema 12.3, p agina 531), se a equa ca o diferencial linear homog enea de segunda ordem y (z ) + a(z )y (z ) + b(z )y (z ) = 0 (13.1) for tal que os coecientes a(z ) e b(z ) s ao fun co es anal ticas de z em torno de um ponto z0 , ent ao suas solu co es ser ao igualmente anal ticas em torno desse ponto e poderemos procurar resolv e-la em termos de s eries de pot encia centradas em z0 :

y (z ) =
n=0

cn (z z0 )n .

(13.2)

O chamado m etodo de s erie de pot encias consiste precisamente em inserir o Ansatz (13.2) na equa ca o (13.1) e determinar recursivamente os coecientes cn . Pelas conclus oes obtidas anteriormente, resumidas no Teorema 12.3 da p agina 531, a solu ca o obtida deve ser convergente pelo menos no maior disco aberto centrado em z0 no qual ambas as fun co es a(z ) e b(z ) sejam tamb em anal ticas. Ilustraremos a aplica ca o desse m etodo na resolu ca o da equa ca o do oscilador harm onico simples e nas equa co es de Legendre, Hermite, Airy e Tchebychev, todas equa co es de interesse em F sica. Ao nal discutiremos a solu ca o do problema geral.

JCABarata. Curso de F sica-Matem atica

Vers ao de 29 de janeiro de 2013.

Cap tulo 13

570/2069

13.1.1

A Equa c ao do Oscilador Harm onico Simples

Por raz oes pedag ogicas, vamos come car discutindo uma equa ca o diferencial bastante simples e familiar. Seja a bemconhecida equa ca o do oscilador harm onico simples
2 y (z ) + 0 y (z ) = 0 ,

(13.3)

2 onde 0 e uma constante. Nesse caso a(z ) = 0 e b(z ) = 0 , ambas anal ticas em toda parte. Procuremos ent ao uma n acil ver que solu ca o da forma y (z ) = n=0 cn z (com z0 = 0). E f

y (z ) =
n=0

ncn z n1 =
n=1

ncn z n1

nn+1

(n + 1)cn+1 z n ,
n=0

ou seja, y (z ) = e que y (z ) =
n=0

(n + 1)cn+1 z n
n=0

(13.4)

n(n + 1)cn+1 z n1 =
n=1

n(n + 1)cn+1 z n1

nn+1

(n + 1)(n + 2)cn+2 z n ,
n=0

ou seja, y (z ) = Inserindo-se (13.4) e (13.5) em (13.3), obtem-se

(n + 1)(n + 2)cn+2 z n .
n=0

(13.5)

2 (n + 1)(n + 2)cn+2 + 0 cn z n = 0 . n=0

Como essa u ltima rela ca o supostamente vale para todo z , tem-se for cosamente que os fatores entre colchetes s ao todos nulos (por que?):
2 (n + 1)(n + 2)cn+2 + 0 cn = 0 ,

ou seja,

cn+2 =

2 0 cn (n + 1)(n + 2)

(13.6)

para todo n 0. A solu ca o dessa u ltima equa ca o recursiva e c2k =


2k (1)k 0 c0 , (2k )!

c2k+1 =

2k (1)k 0 c1 . (2k + 1)!

com k 0. Essas express oes relacionam todos os coecientes cn com os dois primeiros coecientes, c0 e c1 . Inserindo isso na express ao y (z ) =
n n=0 cn z ,

tem-se

y (z ) =
k=0

c2k z 2k +
k=0

c2k+1 z 2k+1 = c0
k=0 k=0

2k (1)k 0 z 2k + c1 (2k )!

k=0

2k (1)k 0 z 2k+1 (2k + 1)!

c0
k=0

(1)k c1 (0 z )2k + (2k )! 0

(1)k (0 z )2k+1 (2k + 1)!

c0 cos(0 z ) +

c1 sen (0 z ) . 0

Na u ltima passagem pudemos identicar as duas s eries de pot encias com as s eries de Taylor (em torno de 0) das fun co es seno e co-seno. Notemos que em problemas menos simples, como os que encontraremos adiante, nem sempre ser a poss vel identicar as s eries resultantes com as s eries de Taylor de fun co es previamente conhecidas, o que nos conduzir a` a deni ca o de novas fun co es, as chamadas fun c oes especiais.

JCABarata. Curso de F sica-Matem atica

Vers ao de 29 de janeiro de 2013.

Cap tulo 13

571/2069

c1 de se notar que a solu E ca o nal, y (z ) = c0 cos(0 z ) + sen (0 z ), e anal tica em toda a parte como fun ca o de z , o 0 que j a era esperado do fato de as fun co es a(z ) e b(z ) serem fun co es anal ticas em toda parte (duas constantes).

Obtivemos, assim, a bem-conhecida solu ca o do oscilador harm onico simples em termos de uma combina ca o linear das fun co es seno e co-seno. Os coecientes c0 e c1 podem ser determinados se mais condi co es forem impostas ` a solu ca o. Por exemplo, se impusermos condi co es iniciais y (0) = y0 e y (0) = v0 , obtemos c0 = y0 e c1 = v0 .

13.1.2

A Equa c ao de Legendre
(1 z 2 )y (z ) 2zy (z ) + ( + 1)y (z ) = 0 (13.7)

A equa ca o diferencial e denominada equa c ao de Legendre1 de ordem2 . Em princ pio, adotamos C, arbitr ario, mas na maioria das aplica co es em F sica apenas valores especiais de s ao considerados, a saber, e tomado um inteiro n ao-negativo. A equa ca o de Legendre e uma parente pr oxima, a equa ca o de Legendre associada, tratada na Se ca o 13.3.1, p agina 616, surgem em v arios problemas de F sica, do Eletromagnetismo ` a Mec anica Qu antica. Tipicamente ambas surgem quando da resolu ca o da equa ca o de Helmholtz pelo m etodo de separa ca o de vari aveis em coordenadas esf ericas em tr es dimens oes. Vide Cap tulo 19, p agina 842. A equa ca o de Legendre acima pode ser posta na forma padr ao (13.1) com a(z ) = 2z 1 z2 e b(z ) = ( + 1) . 1 z2

portanto, leg Claramente, ambas as fun co es s ao anal ticas em um disco de raio 1 centrado em z0 = 0. E, timo procurarmos solu co es na forma y (z ) = n=0 cn z n (com z0 = 0). Tais solu co es ser ao anal ticas pelo menos no disco de raio 1 centrado em z0 = 0. Inserindo-se (13.4)-(13.5) em (13.7), obtem-se
n=0

(n + 1)(n + 2)cn+2 z n

n=0

(n + 1)(n + 2)cn+2 z n+2 2


I

(n + 1)cn+1 z n+1 +( + 1)
n=0 II n=0

cn z n = 0 .

(13.8)

f E acil ver que I

(n + 1)(n + 2)cn+2 z n+2


n=0

nn2

n=2

(n 1)n cn z n =

n=0

(n 1)n cn z n ,

(13.9)

onde, na pen ultima igualdade, zemos a mudan ca de vari aveis n n 2 e, na u ltima, acrescentamos os termos com n = 0 e n = 1 por estes serem nulos. Analogamente,

II

(n + 1)cn+1 z n+1
n=0

nn1

ncn z n =
n=1 n=0

ncn z n ,

(13.10)

onde, na pen ultima igualdade, zemos a mudan ca de vari aveis n n 1 e, na u ltima, acrescentamos o termo com n = 0 por este ser nulo. Assim, (13.8) ca
n=0

(n + 1)(n + 2)cn+2 z n
n=0

n=0

(n 1)n cn z n 2

ncn z n + ( + 1)
n=0 n=0

cn z n = 0 ,

ou seja, (n + 1)(n + 2)cn+2 (n 1)n + 2n ( + 1) cn z n = 0 .

1 Adrien-Marie 2 Aqui

Legendre (17521833). a palavra ordem n ao deve ser confundida com a ordem da equa ca o diferencial, que e dois.

JCABarata. Curso de F sica-Matem atica

Vers ao de 29 de janeiro de 2013.

Cap tulo 13

572/2069

Como (n 1)n + 2n = n(n + 1), obtemos o seguinte conjunto de equa co es (n + 1)(n + 2)cn+2 n(n + 1) ( + 1) cn = 0 , Essas express oes fornecem as seguintes equa co es recursivas para os coecientes cn : cn+2 = n(n + 1) ( + 1) cn , (n + 1)(n + 2) n 0 . (13.11) n 0 .

De maneira an aloga ao que ocorre no caso do oscilador harm onico simples (vide eq. (13.6)), podemos expressar todos os coecientes cn com n par em termos de c0 e todos os coecientes cn com n mpar em termos de c1 . Mais precisamente, tem-se c2k = 1 (2k )!
k 1 l=0

2l(2l + 1) ( + 1) c0 =

( + 1) 2k

k 1 l=1

( + 1) 2l(2l + 1)

c0 ,

c2k+1

1 (2k + 1)!

k 1

l=0

(2l + 1)(2l + 2) ( + 1) c1 =

1 2k + 1

k 1

l=0

( + 1) (2l + 1)(2l + 2)

c1 .

Para C gen erico conclu mos que a solu ca o geral da equa ca o de Legendre e da forma y (z ) = c0 y (z ) + c1 y (z ) , onde
(0) (1)

y (z ) =
k=0 (1) y (z )

(0)

z 2k (2k )!

k 1

l=0

2l(2l + 1) ( + 1)
k 1

(13.12)

=
k=0

z 2k+1 (2k + 1)!

l=0

(2l + 1)(2l + 2) ( + 1)

(13.13)

Conforme comentamos, sabemos a priori que ambas as s eries acima convergem para |z | < 1. O que ocorre caso |z | = 1? Isso e respondido na seguinte proposi ca o, cuja demonstra ca o encontra-se no Ap endice 13.A, p agina 621 (vide tamb em [207] para uma outra prova semelhante): Proposi c ao 13.1 Caso R n ao seja um inteiro n ao-negativo par, a s erie em (13.12) diverge em z = 1. Caso R erie em (13.13) diverge em z = 1. n ao seja um inteiro positivo mpar, a s Essa proposi ca o ensina-nos que as solu co es (13.12) e (13.13) da equa ca o de Legendre ser ao divergentes em z = 1 caso n ao seja um inteiro n ao-negativo e isso para qualquer escolha de c0 e c1 n ao-nulos. Em aplica co es, por em, e muito importante ter-se solu co es nitas no intervalo fechado real [1, 1] de valores de z . A u nica esperan ca que resta reside (0) na situa ca o na qual e um inteiro n ao-negativo e, de fato, podemos vericar que em tal caso y e nita se for par e (1) que y e nita se for mpar. Os polin omios de Legendre

Contemplando a express ao (13.12) facilmente constata-se que no caso em que = 2n, um inteiro n ao-negativo par, tem-se n k 1 z 2k (0) y2n (z ) := 2l(2l + 1) 2n(2n + 1) , (2k )!
k=0 l=0

que e um polin omio de grau 2n em z . Analogamente, contemplando a express ao (13.13) facilmente se constata que no caso em que = 2n + 1, um inteiro positivo mpar, tem-se
n (1) y2n+1 (z ) := k=0

z 2k+1 (2k + 1)!

k 1 l=0

(2l + 1)(2l + 2) (2n + 1)(2n + 2)

JCABarata. Curso de F sica-Matem atica

Vers ao de 29 de janeiro de 2013.

Cap tulo 13

573/2069

que e um polin omio de grau 2n + 1 em z . Assim, vemos que no caso de ser um inteiro n ao-negativo a equa ca o de Legendre tem uma solu ca o nita em toda (0) (1) a parte, a saber, o polin omio c0 y2n (z ), caso = 2n, par, ou o polin omio c1 y2n+1 (z ), caso = 2n + 1, mpar. Denimos, ent ao, m/2 k 1 z 2k (0) c y ( z ) = c 2l(2l + 1) m(m + 1) , m par 0 m 0 (2k )! k =0 l =0 Pm (z ) := . (m1)/2 k 1 z 2k+1 (1) c y ( z ) = c (2l + 1)(2l + 2) m(m + 1) , m mpar 1 1 m (2k + 1)!
k=0 l=0

claro pela deni E ca o acima que Pm e um polin omio de grau m e o coeciente do mon omio de maior grau, z m , vale 1 c0 m! e c1 1 m!
m/21

l=0

2l(2l + 1) m(m + 1) ,

para m par

(m3)/2

l=0

(2l + 1)(2l + 2) m(m + 1) ,

para m mpar.

Por raz oes hist oricas, convenciona-se escolher c0 e c1 de modo que o coeciente do mon omio de maior grau de Pm seja (2m)! igual a 2m . Como facilmente se constata ap o s alguns c a lculos entediantes, isso conduz ` a seguinte express ao para os 2 (m!) polin omios Pm (z ):
m/2

Pm (z ) :=
a=0

(1)a (2m 2a)! z m2a , 2m (m a)! (m 2a)! a!

(13.14)

A prova de (13.14) pode ser encontrada no Ap endice 13.B, p agina 622. E. 13.1 Exerc cio. Tente provar (13.14) sem ler o Ap endice 13.B.

onde m/2 e o maior inteiro menor ou igual a m/2, ou seja, m, m par, 2 m := 2 m1 , m mpar. 2

A express ao (13.14) dene os assim denominados polin omios de Legendre de grau m, cada qual e solu ca o da equa ca o de Legendre de ordem m (1 z 2 )y (z ) 2zy (z ) + m(m + 1)y (z ) = 0 , com m inteiro n ao-negativo. Como comentamos, essa equa ca o possui, para cada m inteiro n ao-negativo, uma segunda solu ca o que e, por em, divergente para z 1. Os quatro primeiros polin omios de Legendre s ao P0 (z ) = 1 , P1 (z ) = z , 1 3 P2 (z ) = + z 2 , 2 2 3 5 P3 (z ) = z + z 3 , 2 2

como facilmente se v e pela deni ca o acima. Os polin omios de Legendre possuem v arias propriedades importantes, tais como rela co es de ortogonalidade, f ormulas de recorr encia etc., as quais ser ao discutidas na Se ca o 14.2.1, p agina 642. Tamb em remetemos o estudante ` a literatura pertinente supracitada. A Figura 13.1, p agina 574, exibe o gr aco dos primeiros polin omios de Legendre no intervalo [1, 1].

JCABarata. Curso de F sica-Matem atica

Vers ao de 29 de janeiro de 2013.

Cap tulo 13

574/2069

P 0
1.0 0.8

P 1
0.6 0.4 0.2 0.0 1.0 0.8 0.6 0.4 0.2 0.2 0.4 0.6 0.8 1.0 0.0 0.2 0.4 0.6 0.8 1.0

P 2 P 3 P 4

Figura 13.1: Os polin omios de Legendre P0 a P4 no intervalo [1, 1].

13.1.3

A Equa c ao de Hermite
y (z ) 2zy (z ) + y (z ) = 0,
3

A equa ca o diferencial (13.15) com C e denominada equa c ao de Hermite . Essa equa ca o e famosa por surgir em um problema b asico da Mec anica Qu antica, a saber, o problema do oscilador harm onico unidimensional. Vide Se ca o 19.7, p agina 907. A rela ca o de (13.15) com a equa ca o hipergeom etrica conuente e exibida na Se ca o 13.2.8, p agina 614. Comparando ` a forma padr ao (13.1), constatamos que aqui a(z ) = 2z e b(z ) = .

Ambas essas fun co es s ao anal ticas em todo o plano complexo e, pelo Teorema 12.3 da p agina 531, assim ser ao as solu co es da equa ca o de Hermite, sendo que podemos encontr a-las atrav es de uma expans ao em s erie de pot encias em torno de z0 = 0: y (z ) = n=0 cn z n . Inserindo-se (13.4)-(13.5) em (13.15), obtem-se
n=0

(n + 1)(n + 2)cn+2 z n 2

(n + 1)cn+1 z n+1 +
n=0 II n=0

cn z n = 0 .

(13.16)

A soma II pode ser escrita como em (13.10) e, assim, (13.16) ca


n=0
3 Charles

(n + 1)(n + 2)cn+2 z 2

ncn z +
n=0 n=0

cn z n = 0 ,

Hermite (18221901).

JCABarata. Curso de F sica-Matem atica

Vers ao de 29 de janeiro de 2013.

Cap tulo 13

575/2069

ou seja,

n=0

(n + 1)(n + 2)cn+2 + ( 2n) cn z n = 0 ,

para todo z C, o que implica (n + 1)(n + 2)cn+2 + ( 2n) cn = 0, n 0. Disso conclu mos que cn+2 = 2n cn , (n + 1)(n + 2) n0. (13.17)

Assim como no caso do oscilador harm onico simples e no caso da equa ca o de Legendre, os coecientes cn com n par s ao proporcionais a c0 e os coecientes cn com n mpar s ao proporcionais a c1 . Mais precisamente, tem-se c2 = c0 , 2 c2k = c0 (2k )!
k 1 l=1

(4l ) ,

k2,

c2k+1

= c1

1 (2k + 1)!

l=1

(4l 2 ) ,

k1.

Desta forma, chegamos ` a seguinte solu ca o geral da equa ca o de Hermite: y (z ) = c0 y (z ) + c1 y (z ) , onde


(0) y (z ) := 1 (0) (1)

2 z 2

k=2

z 2k (2k )!

k 1

l=1

(4l ) ,

(1) y (z ) := z + k=1

z 2k+1 (2k + 1)!

l=1

(4l 2 ) .

Conforme comentamos, o Teorema 12.3 da p agina 531 garante-nos que ambas as s eries acima convergem absolutamente (0) (1) para todo z C, fazendo de y e y fun co es inteiras de z . Os polin omios de Hermite Vamos agora passar ` a deni ca o dos chamados polin omios de Hermite. Nestas notas usamos a chamada deni ca o f sica dos polin omios de Hermite. H a uma outra conven ca o, usada especialmente na Teoria das Probabilidades, que difere da deni ca o usada em F sica por um reescalonamento. O leitor deve, por isso, ter cuidado ao comparar nossas express oes com outras usadas em textos da Teoria das Probabilidades.

No caso em que z e restrita a ser uma vari avel real, cham emo-la x, e poss vel demonstrar que se for real e as s eries acima forem innitas, ent ao ambas comportam-se, para |x| grande, como fun co es que crescem mais r apido que exp(x2 /2). Isso e provado no Ap endice 13.C, p agina 624, e, por outros meios, em [151] ou em [145]. No contexto da Mec anica Qu antica esse fato e indesejado, pois conduz a fun co es de onda que n ao s ao de quadrado integr avel (vide Se ca o 19.7, p agina 907). Assim, interessa-nos investigar sob quais circunst ancias as s eries acima podem ser reduzidas a polin omios. Como vemos facilmente por (13.17), isso se d a apenas quando for um n umero inteiro n ao-negativo e par: = 2m, com m = 0, 1, 2, . . . etc. De fato, se = 2m, com m = 0, 1, 2, . . . etc., a express ao (13.17) diz-nos que 0 = cm+2 = (0) (1) cm+4 = cm+6 = etc. Assim, caso m for par, y ser a um polin omio de ordem m e caso m for mpar, y ser a um polin omio de ordem m. Dena-se, assim, (2)m/2 (m 1)!! y2m (z ),
(0)

para m par, (13.18)

Hm (z ) :=

(2)(m+1)/2 (m!!) y2m (z ), para m mpar,

(1)

JCABarata. Curso de F sica-Matem atica

Vers ao de 29 de janeiro de 2013.

Cap tulo 13

576/2069

ou seja, m 2 2k k1 2m 2 z (2)m/2 (m 1)!! 1 z 2m (4l 2m) , para m par, 2 (2 k )! l =1 k =2 (2)(m+1)/2 (m!!) z +


m 1 2

Hm (z ) :=

(13.19)

k=1

z 2k+1 (2k + 1)!

l=1

De maneira compacta, podemos escrever isso da seguinte forma


m/2

(4l 2(m + 1)) ,

para m mpar.

Hm (z ) :=
k=0

(1)k m! (2z )m2k . k ! (m 2k )!

(13.20)

A demonstra ca o pode ser encontrada no Ap endice 13.D, p agina 625. endice 13.D. E. 13.2 Exerc cio. Tente mostrar isso sem ler o Ap As fun co es Hm (z ) s ao polin omios de grau m e s ao denominados polin omios de Hermite. Os fatores (2)m/2 (m 1)!! e (2)(m+1)/2 (m!!) prov em de uma conven ca o hist orica sobre a normaliza ca o dos polin omios de Hermite. Os quatro primeiros s ao H0 (z ) = 1 , H1 (z ) = 2z , H2 (z ) = 2 + 4z 2 , H3 (z ) = 12z + 8z 3 ,

como facilmente se v e pela deni ca o acima. Cada polin omio de Hermite Hm e solu ca o da equa ca o de Hermite y (z ) 2zy (z ) + 2my (z ) = 0, com m inteiro positivo. Como mencionamos, essa equa ca o possui ainda uma segunda solu ca o que, embora nita para todo z C, cresce muito rapidamente quando z e real e |z | , o que elimina seu interesse no contexto da Mec anica Qu antica (especicamente, no problema do oscilador harm onico). Os polin omios de Hermite possuem v arias propriedades importantes, tais como rela co es de ortogonalidade, f ormulas de recorr encia etc., que ser ao discutidas na Se ca o 14.2.3, p agina 658. Tamb em remetemos o estudante ` a literatura pertinente supracitada.

13.1.4

A Equa c ao de Airy
y (z ) zy (z ) = 0

A equa ca o diferencial e denominada equa c ao de Airy4 . Essa equa ca o surge em v arios contextos, como por exemplo no estudo da propaga ca o de ondas eletromagn eticas em meios com ndice de refra ca o vari avel, no estudo da reex ao de ondas de radio na atmosfera e, de especial import ancia, na Mec anica Qu antica, mais especicamente, na equa ca o de Schr odinger de uma part cula que se move em uma dimens ao sob um potencial que cresce linearmente com a posi ca o (i.e., sob uma for ca constante). Na Se ca o 19.5.3, p agina 900, tratamos com detalhe de um outro problema f sico onde ocorre a equa ca o de Airy, a saber, o problema de determinar os modos de vibra ca o de uma corda n ao-homog enea cuja densidade varia linearmente com a posi ca o. Comparando ` a forma padr ao (13.1), constatamos que aqui a(z ) = 0 e b(z ) = z . Ambas essas fun co es s ao anal ticas em todo o plano complexo e, pelo Teorema 12.3 da p agina 531, assim ser ao as solu co es da equa ca o de Airy, sendo que podemos encontr a-las atrav es de uma expans ao em s erie de pot encias em torno de z0 = 0: y (z ) = n=0 cn z n .
4 George Biddell Airy (18011892). A equa ca o de Airy surgiu originalmente em seus estudos sobre a Teoria do Arco- Iris. Vide tamb em On the diraction of an object-glass with circular aperture, G. B. Airy, in Transactions of the Cambridge Philosophical Society (1835).

JCABarata. Curso de F sica-Matem atica

Vers ao de 29 de janeiro de 2013.

Cap tulo 13

577/2069

Inserindo-se (13.5) em (13.15), obtem-se


n=0

(n + 1)(n + 2)cn+2 z n

cn z n+1 = 0 .
n=0 III

(13.21)

A express ao III pode ser escrita como


III =
n=0

cn z n+1 =
n=1

cn1 z n

pela mudan ca n n 1. Assim, a equa ca o de Airy diz-nos que


n=0

(n + 1)(n + 2)cn+2 z n

cn1 z n = 0 ,
n=1

ou seja, 2 c2 +

n=1

(n + 1)(n + 2)cn+2 cn1 z n = 0 . n1. (13.22)

Com isso, devemos ter c2 = 0 , ou seja, c2 = 0 , cn+3 = (n + 1)(n + 2)cn+2 cn1 = 0, cn , (n + 2)(n + 3)

n0.

O conjunto de coecientes {cn , n = 0, 1, 2, . . .} e a uni ao dos seguintes tr es conjuntos disjuntos: {c3k , k = 0, 1, 2, . . .} = {c3k+1 , k = 0, 1, 2, . . .} = {c3k+2 , k = 0, 1, 2, . . .} = { c 0 , c 3 , c 6 , c 9 , . . .} {c1 , c4 , c7 , c10 , . . .} {c2 , c5 , c8 , c11 , . . .}

As rela co es de recorr encia de (13.22) implicam que os coecientes do primeiro conjunto acima s ao proporcionais a c0 , que os coecientes do segundo conjunto acima s ao proporcionais a c1 e que os coecientes do terceiro conjunto acima s ao proporcionais a c2 . Por em, como c2 = 0, conclu mos que os coecientes do terceiro conjunto s ao todos nulos. Logo,

y (z ) =
k=0

c3k z 3k +
k=0

c3k+1 z 3k+1 .

As rela co es de recorr encia de (13.22) dizem-nos que c3k = 1 c0 , 3k k ! (3k 1)!!!

c3k+1 =

1 c1 3k k ! (3k + 1)!!!

c3k+2 = 0 ,

para todo k 0. Assim, a solu ca o geral da equa ca o de Airy e y (z ) = c0


k=0

z 3k 3k k ! (3k 1)!!!

+ c1
k=0

z 3k+1 3k k ! (3k + 1)!!!

(13.23)

Como 3k k ! = (3k )!!! (por que?), podemos reescrever isso como

y (z ) = c0
k=0

z 3k (3k )!!! (3k 1)!!!

+ c1
k=0

z 3k+1 (3k )!!! (3k + 1)!!!

JCABarata. Curso de F sica-Matem atica

Vers ao de 29 de janeiro de 2013.

Cap tulo 13

578/2069

As fun co es de Airy de primeiro e de segundo tipo 3k k + 2 3


2 3

H a ainda uma outra maneira de reescrever (13.23), a saber, usando as identidades (3k 1)!!! = , (3k + 1)!!! = 3k k + 4 3
4 3

(13.24)

sendo, para x 0,

(x) :=
0

et tx1 dt

(13.25)

a bem conhecida Fun c ao Gama de Euler, a qual satisfaz (x + 1) = x(x) . assim como a assim denominada f ormula de duplica c ao (x)(x + 1/2) = 212x (2x) . A Fun ca o Gama de Euler e suas propriedades s ao discutidas com mais detalhe no Cap tulo 7, p agina 270. E. 13.3 Exerc cio. Usando (13.26) prove (13.24). Com isso, podemos escrever a solu ca o (13.23) da equa ca o de Airy como y (z ) = c0 2 3 z 3k 2 k 3 k! k + k=0

(13.26)

(13.27)

2 3

+ c1

4 3

k=0

32k

z 3k+1 k! k +

4 3

(13.28)

Essa express ao pode ser escrita como combina ca o linear das seguintes fun co es: Ai(z ) := z 3k 2 k +2 / 3 3 k! k + k=0
2 3

k=0

32k+4/3

z 3k+1 k! k +

4 3

(13.29)

Bi(z ) :=

31/2
k=0

z 3k 32k+2/3 k ! k +

2 3

+
k=0

z 3k+1 32k+4/3 k ! k +

4 3

(13.30)

as quais s ao denominadas fun c oes de Airy de primeiro tipo e de segundo tipo, respectivamente. As fun co es Ai(z ) e Bi(z ) foram denidas como acima por conven ca o hist orica. Ambas s ao anal ticas para todo z C e representam solu co es da equa ca o de Airy. Propriedades dessas fun co es podem ser estudadas em [151]. Como veremos com um pouco mais de detalhe ` a p agina 606, a equa ca o de Airy pode ser transformada em uma equa ca o de Bessel de ordem 1/3 e as fun co es de Airy Ai(z ) e Bi(z ) podem ser escritas em termos das fun co es de Bessel J1/3 . Vide express oes (13.127) e (13.128).

13.1.5

A Equa c ao de Tchebychev
(1 z 2 )y (z ) z y (z ) + 2 y (z ) = 0 (13.31)

A equa ca o diferencial e denominada equa c ao de Tchebychev5 . Em princ pio adotamos C arbitr ario, mas o maior interesse estar a no caso em que e um inteiro n ao-negativo. A equa ca o de Tchebychev acima pode ser posta na forma padr ao (13.1) com a(z ) =
5 Pafnuty

z 1 z2

b(z ) =

2 . 1 z2

Lvovich Tchebychev (18211894).

JCABarata. Curso de F sica-Matem atica

Vers ao de 29 de janeiro de 2013.

Cap tulo 13

579/2069

portanto, leg Claramente, ambas as fun co es s ao anal ticas em um disco de raio 1 centrado em z0 = 0. E, timo procurarmos co es ser ao anal ticas pelo menos no disco de raio 1 centrado solu co es na forma y (z ) = n=0 cn z n (com z0 = 0). Tais solu em z0 = 0. Inserindo-se (13.4)-(13.5) em (13.31), obtem-se
n=0

(n + 1)(n + 2)cn+2 z n

n=0

(n + 1)(n + 2)cn+2 z n+2


I

(n + 1)cn+1 z n+1 +2
n=0 II n=0

cn z n = 0 .

(13.32)

Novamente, I e II s ao dadas como em (13.9) e (13.10), respectivamente, e, portanto, (13.32) ca


n=0

(n + 1)(n + 2)cn+2 z n

n=1

(n 1)n cn z n

ncn z n + 2
n=1 n=0

cn z n = 0 ,

ou seja, 2c2 + 2 c0 +
n=1 2

(n + 1)(n + 2)cn+2 (n 1)n + n 2 cn z n = 0 .

Como (n 1)n + n = n , obtemos o seguinte conjunto de equa co es 2c2 + 2 c0 (n + 1)(n + 2)cn+2 n2 2 cn n2 2 cn , (n + 1)(n + 2) = 0, = 0, n 1 .

Essas express oes fornecem as seguintes equa co es recursivas para os coecientes cn : cn+2 = n 0 . (13.33)

De maneira an aloga ao que zemos em exemplos anteriores, podemos expressar todos os coecientes cn com n par em termos de c0 e todos os coecientes cn com n mpar em termos de c1 . Mais precisamente, tem-se c2k = 1 (2k )!
k 1

l=0

(2l)2 2 c0 ,

c2k+1

1 (2k + 1)!

k 1

l=0

(2l + 1)2 2 c1 .

Para C gen erico conclu mos que a solu ca o geral da equa ca o de Tchebychev e da forma y (z ) = c0 y (z ) + c1 y (z ) , onde
(0) (1)

y (z ) = 1 +
k=1

(0)

z 2k (2k )!

k 1 l=0

(2l)2 2 ,
k 1

(13.34)

y (z ) = z +
k=1

(1)

z 2k+1 (2k + 1)!

l=0

(2l + 1)2 2 .

(13.35)

Os polin omios de Tchebychev

Como mencionamos, o principal interesse reside no caso em que e um inteiro n ao-negativo: = m. Nesse caso e (0) (1) f acil ver que ym (z ) ser a um polin omio de grau m, caso m seja par e ym (z ) ser a um polin omio de grau m, caso m seja

JCABarata. Curso de F sica-Matem atica

Vers ao de 29 de janeiro de 2013.

Cap tulo 13

580/2069

mpar. Esses polin omios s ao


m/2 (0) ym (z ) =

1+
k=1

z 2k (2k )!

k 1 l=0

(2l)2 m2 , m par,

(13.36) (13.37)

(m1)/2 (1) (z ) = ym

z+
k=1

z 2k+1 (2k + 1)!

k 1

l=0

mpar. (2l + 1)2 m2 , m

(13.38)

Por uma conven ca o hist orica, costuma-se redenir esses polin omios multiplicando-os por uma constante dependente de m de modo a fazer o coeciente do mon omio de maior grau, z m , igual a 2m1 . Ap os alguns c alculos entediantes (indicados no Ap endice 13.F, p agina 628) o estudante poder a convencer-se que, com essa conven ca o, os polin omios acima podem ser escritos de uma forma compacta como T0 (z ) = 1 , m Tm (z ) := 2
m/2

k=0

(1)k (m k 1)! (2z )m2k , k ! (m 2k )!

m>0.

(13.39)

Os polin omios assim denidos s ao denominados polin omios de Tchebychev, os quais desempenham um papel central na Teoria da Aproxima c ao6 . Vide, por exemplo, [54], [241], [214] ou [169]. Os quatro primeiros polin omios de Tchebychev s ao T0 (z ) = 1 , T1 (z ) = z , T2 (z ) = 2z 2 1 , T3 (z ) = 4z 3 3z .

poss E vel ainda demonstrar que os polin omios de Tchebychev podem ser escritos na forma
m/2

Tm (z ) =
p=0

(1)p

m 2p

z m2p 1 z 2

(13.40)

v alida para todo m = 0, 1, 2, 3, 4, . . .. Uma das mais curiosas e importantes propriedades dos polin omios de Tchebychev Tm e a seguinte identidade: Tm (x) = cos m arccos(x) . (13.41)

v alida para 1 x 1. Essa rela ca o pode ser demonstrada constatando-se que o lado direito e solu ca o da equa ca o de Tchebychev e constatando-se que o lado direito e um polin omio (o que e um tanto uma surpresa. Vide abaixo) de grau m e que o coeciente de seu termo de maior grau em z e 2 m1 . A rela ca o (13.41) pode ser facilmente demonstrada a partir da express ao (13.40) (ou vice-versa). Vide Exerc cio E. 13.4, abaixo. Alguns autores adotam (13.41) como deni ca o dos polin omios de Tchebychev.
6 Para uma discuss ao sobre o interessante problema de Engenharia que conduziu Tchebychev aos polin omios que levam seu nome, vide [139].

JCABarata. Curso de F sica-Matem atica

Vers ao de 29 de janeiro de 2013.

Cap tulo 13

581/2069

ao: denindo y arccos(x), tem-se E. 13.4 Exerc cio resolvido. Prove (13.41) para 1 x 1. Sugest cos m arccos(x) = = = = cos(my ) 1 imy e + eimy 2 1 [(cos y + i sen y )m + (cos y i sen y )m ] 2 1 2 1 2 x + i 1 x2
m q=0 m

+ x i 1 x2 1 x2
q m

m q

xmq i

+
q=0

m q

xmq i

1 x2

m/2

=
p=0

(1)p

m 2p

xm2p 1 x2

(13.40)

Tm (x) ,

que e o que quer amos. Na passagem indicada por usamos o fato que os termos com q mpar nas duas somas anteriores cancelam-se mutuamente, sobrando, portanto, apenas os termos com q par, ou seja, da forma q = 2p com p = 0, . . . , m/2. Para provar (13.41) a partir de (13.40), basta ler as linhas acima do m para o come co. De (13.41) segue imediatamente a interessante propriedade de composi ca o Tn (Tm (x)) = Tnm (x) , (13.42)

v alida para todos n, m inteiros n ao-negativos. Como ambos os lados de (13.42) s ao polin omios, essa rela ca o vale para todo x complexo.

13.1.6

O Caso de Equa c oes Regulares Gerais


y (z ) + a(z )y (z ) + b(z )y (z ) = 0 (13.43)

Nas p aginas acima resolvemos em v arios exemplos particulares a equa ca o

em casos em que os coecientes a(z ) e b(z ) s ao fun co es anal ticas de z em torno de um ponto z0 . Para tal, evocando o Teorema 12.3, p agina 531, procuramos solu co es na forma de s eries de pot encias:

y (z ) =
n=0

cn (z z0 )n .

(13.44)

Vamos agora mostrar como o m etodo que descrevemos se aplica ao caso geral no qual as fun co es a(z ) e b(z ) s ao tamb em dadas em termos de s eries de pot encias:

a(z ) =
n=0

an (z z0 )n ,

b(z ) =
n=0

bn (z z0 )n .

Usando novamente (13.4) e (13.5) a equa ca o (13.43) ca (adotamos daqui para frente z0 = 0, sem perda de generalidade)

(n + 1)(n + 2)cn+2 z n +
n=0 n=0

an z n

(n + 1)cn+1 z n
n=0

+
n=0

bn z n
n=0

cn z n

(13.45)

JCABarata. Curso de F sica-Matem atica

Vers ao de 29 de janeiro de 2013.

Cap tulo 13

582/2069

Para o produto de duas s eries de pot encia


p=0

p z p e

q=0

q z q vale
n

p z p
p=0 q=0

q z q

=
p=0 q=0

p q z p+q =
n=0 m=0

nm m

zn .

(13.46)

E. 13.5 Exerc cio. Mostre isso. Assim, (13.45) ca


n n

(n + 1)(n + 2)cn+2 z n +
n=0 n=0 m=0

anm (m + 1)cm+1

zn +
n=0 m=0

bnm cm

z n = 0,

ou seja,

(n + 1)(n + 2)cn+2 +
n=0

(m + 1)anm cm+1 +
m=0 n m=0

bnm cm z n = 0,

o que implica cn+2 =

1 (m + 1)anm cm+1 + bnm cm (n + 1)(n + 2) m=0

(13.47)

para todo n 0. Observe que essa express ao determina cn+2 em termos de c0 , c1 , . . . , cn+1 . Assim, apenas xando c0 e c1 podemos determinar todos os demais coecientes cn atrav es da express ao recursiva acima. Como dissemos, os resultados que nos conduziram ao Teorema 12.3, p agina 531, garantem-nos que a s erie y (z ) = assim obtida e convergente na mesma regi ao em que convergem as s eries de a(z ) e b(z ), de modo que n ao precisamos provar isso. Alguns autores (por exemplo, [207]) usam as express oes recursivas (13.47) para demonstrar a ao e mais necess ario, mas o estudante converg encia da s erie y (z ) = n=0 cn z n . Como dissemos, pelo nosso proceder isso n interessado e convidado a estudar essa outra (elegante) demonstra ca o no texto supracitado.
n n=0 cn z

Para futura refer encia, resumimos nossas conclus oes sobre equa co es regulares no seguinte teorema. Teorema 13.1 (Solu c ao de equa co es regulares por expans ao em s erie de pot encias) Considere-se a equa c ao diferencial y (z ) + a(z )y (z ) + b(z )y (z ) = 0 , (13.48) z C, com a(z ) e b(z ) anal ticas em torno de z0 e expressas em termos de suas s eries de Taylor em torno de z0 como

a(z ) =
n=0

an (z z0 )n ,

b(z ) =
n=0

bn (z z0 )n ,

s eries estas supostas absolutamente convergentes em |z z0 | < r, para algum r > 0. Ent ao a solu c ao geral da equa c ao (13.48) pode ser expressa em termos de uma expans ao em s erie de pot encias em z z0 :

y (z ) =
n=0

cn (z z0 )n ,

onde os coecientes cn podem ser obtidos atrav es das rela c oes recursivas cn+2 1 (m + 1)anm cm+1 + bnm cm , = (n + 1)(n + 2) m=0
n

n0,

a partir dos dois primeiros coecientes c0 e c1 , arbitr arios. A expans ao em s erie de pot encias para y (z ) converge absolutamente pelo menos na regi ao |z z0 | < r, onde representa uma fun c ao anal tica.

JCABarata. Curso de F sica-Matem atica

Vers ao de 29 de janeiro de 2013.

Cap tulo 13

583/2069

13.2

Solu c ao de Equa co es Singulares Regulares. O M etodo de Frobenius

Na presente se ca o ilustraremos o Teorema 12.4, p agina 535, estudando a solu ca o, por um m etodo devido a Frobenius7 , de algumas equa co es diferenciais ordin arias, homog eneas de segunda ordem e singulares regulares de interesse (especialmente em F sica). Boa parte dos m etodos apresentados nos exemplos aplicam-se a equa co es de ordem maior que dois, mas n ao trataremos de tais generaliza co es aqui pois elas pouco apresentam de especial e seu interesse na F sica e reduzido. Vale aqui novamente a advert encia sobre a omiss ao de alguns detalhes de c alculos, sendo o estudante novamente convidado a complet a-los como exerc cio (todos merecem ser feitos ao menos uma vez na vida). Todas as equa co es particulares tratadas e suas solu co es s ao amplamente discutidos na vasta literatura pertinente, por exemplo, aquela listada ` a p agina 569. Conforme demonstramos em p aginas anteriores (Teorema 12.3, p agina 531), se a equa ca o diferencial linear homog enea de segunda ordem a(z ) b(z ) y (z ) + y (z ) = 0 (13.49) y (z ) + (z z0 ) (z z0 )2 for tal que a(z ) e b(z ) s ao fun co es anal ticas de z em torno de um ponto z0 , ent ao o coeciente
b(z ) (z z 0 )2 a( z ) (z z 0 )

tem no m aximo

uma singularidade de tipo polo de ordem 1 em z0 e o coeciente tem no m aximo uma singularidade de tipo polo de ordem 2 em z0 . Assim, pelas nossas deni co es pr evias, z0 e um ponto singular regular da equa ca o (13.49). Nesse caso, o Teorema 12.3, p agina 531, diz-nos que ou a equa ca o (13.49) tem duas solu co es independentes da forma

y (z ) = (z z0 )

n=0

cn (z z0 )n .

(13.50)

e absolutamente convergente para |z z0 | < r (e, portanto, representa uma fun ca o onde C e a s erie n=0 cn (z z0 )n anal tica em torno de z0 ) ou ent ao a equa ca o (13.49) tem duas solu co es independentes, uma da forma (13.50) e outra da forma

y (z ) = (z z0 ) (ln(z z0 ))

n=0

cn (z z0 )n + (z z0 )

n=0

vn (z z0 )n .

(13.51)

ao absolutamente convergentes para |z z0 | < r (e, onde, novamente as s eries n=0 cn (z z0 )n e n=0 vn (z z0 )n s portanto, representam fun co es anal ticas em torno de z0 ). Em ambos os casos acima r > 0 e o raio do maior disco aberto centrado em z0 dentro do qual a(z ) e b(z ) s ao anal ticas. O chamado m etodo de Frobenius consiste precisamente em inserir-se o Ansatz (13.50) na equa ca o (13.49) e determinar recursivamente os coecientes cn , assim como o expoente . Caso duas solu co es distintas sejam encontradas dessa forma, o problema est a resolvido. Caso se encontre apenas uma solu ca o, ent ao uma segunda solu ca o da forma (13.51) deve ser procurada atrav es da determina ca o recursiva dos coecientes cn e vn , assim como dos expoentes e . Ao contr ario do que zemos no caso de equa co es regulares, quando primeiro exploramos exemplos particulares para depois tratarmos do caso geral, e mais conveniente no presente contexto que nos apoderemos primeiramente da an alise geral para depois tratarmos de equa co es espec cas, pois uma vis ao pr evia das complica co es envolvidas nos auxiliar aa evitar certas armadilhas ocultas no tratamento de equa co es singulares regulares particulares8 . Ilustraremos o m etodo de Frobenius apresentando a resolu ca o da equa ca o de Euler, da equa ca o de Bessel, da equa ca o de Laguerre e das equa co es hipergeom etrica e hipergeom etrica conuente, todas de interesse em F sica. O principal teorema que demonstraremos, o qual resume os resultados do m etodo de Frobenius e expressa a solu ca o de uma equa ca o singular regular homog enea de segunda ordem geral, e o seguinte: Teorema 13.2 (Solu c ao de equa co es singulares regulares pelo m etodo de Frobenius) Seja a equa c ao diferencial (z z0 )2 y (z ) + (z z0 )a(z )y (z ) + b(z )y (z ) = 0 , (13.52)
Georg Frobenius (18491917). estudante e convidado a n ao entrar em p anico diante da aparente complexidade de algumas express oes que obteremos. Na maioria das equa co es diferenciais de interesse as fun co es a(z ) e b(z ) s ao apenas polin omios de grau 0, 1 ou 2 e as express oes obtidas no tratamento geral se simplicam um tanto.
8O 7 Ferdinand

JCABarata. Curso de F sica-Matem atica

Vers ao de 29 de janeiro de 2013.

Cap tulo 13

584/2069

z C, com a(z ) e b(z ) anal ticas em torno de z0 e expressas em termos de suas s eries de Taylor em torno de z0 como

a(z ) =
n=0

an (z z0 )n ,

b(z ) =
n=0

bn (z z0 )n ,

s eries estas supostas absolutamente convergentes em |z z0 | < r, para algum r > 0. Seja denido o polin omio de segundo grau f (x) := x(x 1) + a0 x + b0 = x2 + (a0 1)x + b0 , e considere-se a equa c ao alg ebrica f (x) = 0 , a qual e denominada equa ca o indicial. Sejam as solu c oes dessa equa c ao no plano complexo: (a0 1)2 4b0 1 a0 + (a0 1)2 4b0 e + = . 2 2 Ent ao a equa c ao (13.52) possui duas solu c oes independentes y1 (z ) e y2 (z ), v alidas pelo menos na regi ao 0 < |z z0 | < r. A forma dessas solu c oes varia conforme as seguintes condi c oes complementares sobre e + : 1. + Z, 2. + = 0 ou 3. + Z \ {0}, como enumeramos a seguir: = 1 a0 1. Caso + Z. Nesse caso tem-se

(13.53)

y1 (z ) = (z z0 ) onde

n=0

cn ( )(z z0 )n

y2 (z ) = (z z0 )+

n=0

cn (+ )(z z0 )n ,

(13.54)

1 cn ( ) = f ( + n)

n1

(m + )anm + bnm cm ( ) ,
m=0

(13.55)

para todo n 1. Essas express oes recursivas permitem-nos obter todos os cn ( ) a partir de um c0 ( ) n ao-nulo arbitr ario e, respectivamente, todos os cn (+ ) a partir de um c0 (+ ) n ao-nulo arbitr ario. 2. Caso + = 0. Neste caso (a0 1)2 4b0 = 0 e = + = 0 com 0 := e tem-se

1 a0 2

y1 (z ) = (z z0 )0 onde

n=0

cn (0 ) (z z0 )n

y2 (z ) = y1 (z ) ln(z z0 ) + (z z0 )0
n1

n=0

vn (0 ) (z z0 )n , (13.56)

cn (0 ) = para todo n 1, e vn (0 ) = 1 f (0 + n)

1 f (0 + n)

(m + 0 )anm + bnm cm (0 )
m=0

(13.57)

2(n + 0 ) 1 cn (0 ) +

anm cm (0 )
m=0 n1

+
m=0

(m + 0 )anm + bnm vm (0 ) ,

n 1 , (13.58)

onde os coecientes cn (0 ) s ao obtidos recursivamente a partir de um c0 (0 ) n ao-nulo arbitr ario e os coecientes vn (0 ) s ao obtidos recursivamente a partir dos coecientes cm (0 ) e a partir de um v0 (0 ) arbitr ario (mas que pode ser escolhido igual a zero).

JCABarata. Curso de F sica-Matem atica

Vers ao de 29 de janeiro de 2013.

Cap tulo 13

585/2069

3. Caso + Z \ {0}. Neste caso + =

(a0 1)2 4b0 e um inteiro n ao-nulo. Denamos ent ao n0 = (a0 1)2 4b0 .

Claro est a que n0 {1, 2, 3, 4, . . .}. Denamos tamb em 1 := , 2 := + , caso + 1, ou (13.59) 1 := + , Com essas deni c oes tem-se 1 = 2 + n 0 . Ent ao,

2 := ,

caso + 1.

y1 (z ) = (z z0 )1 onde

n=0

cn (1 )(z z0 )n

y2 (z ) = Ay1 (z ) ln(z z0 ) + (z z0 )2
n1

n=0

vn (z z0 )n ,

(13.60)

cn (1 ) = para n 1 e 1 f ( + n) 2 vn = arbitr ario , 1 f ( + n) 2

1 (m + 1 )anm + bnm cm (1 ) , f (1 + n) m=0

(13.61)

n1

(m + 2 )anm + bnm vm ,
m=0

para 1 n n0 1 ,

para n = n0 ,

(13.62)

n1

Agnn0 +
m=0

(m + 2 )anm + bnm vm

para n > n0 ,

onde,

A = e

1 c0 (1 ) n0

n0 1

[(m + 2 )an0 m + bn0 m ] vm


m=0 n

(13.63)

gn = [2(n + 1 ) 1] cn (1 ) +

anm cm (1 ) ,
m=0

n0.

(13.64)

As express oes recursivas para cn (1 ) dependem de um c0 (1 ) n ao-nulo e arbitr ario e as express oes recursivas para vn dependem tamb em de um v0 arbitr ario. Todas as s eries de pot encia em z z0 apresentadas acima convergem absolutamente pelo menos na regi ao |z z0 | < r e nela representam, portanto, fun c oes anal ticas. Para a demonstra ca o desse teorema devotaremos toda a Se ca o 13.2.1. Em uma primeira leitura o estudante poder a dispensar-se de um estudo detalhado da demonstra ca o e passar mais rapidamente aos exemplos discutidos na Se ca o 13.2.2, p agina 593, e seguintes.

JCABarata. Curso de F sica-Matem atica

Vers ao de 29 de janeiro de 2013.

Cap tulo 13

586/2069

13.2.1

Equa c oes Singulares Regulares. O Caso Geral

Daqui para frente, sem perda de generalidade, adotaremos z0 = 0. Seja, ent ao, a equa ca o (13.49) escrita agora na forma z 2 y (z ) + za(z )y (z ) + b(z )y (z ) = 0 (13.65)

com a(z ) e b(z ) anal ticas em torno de z0 = 0 e expressas em termos de suas s eries de Taylor em torno de 0 como

a(z ) =
n=0

an z n ,

b(z ) =
n=0

bn z n .

Sob a luz do Teorema 12.4, p agina 535, procuraremos primeiramente uma solu ca o na forma

y (z ) =
n=0

cn z n+ .

(13.66)

Antes de iniciarmos nossa an alise, comentemos que, sem perda de generalidade, podemos sempre adotar o primeiro coeciente, c0 , como n ao-nulo: c0 = 0. Isso se deve ao seguinte. Se cm fosse o primeiro coeciente n ao-nulo, ter amos

y (z ) =
n=m

cn z n+ .

Agora, com a mudan ca de vari avel n = n m car amos com

y (z ) =
n =0 redenindo c amos com n := cn +m e = + m, car

cn +m z n +( +m)

n + c = n z

y (z ) =
n =0

n+ c . nz n=0

Au ltima express ao possui a mesma estrutura de (13.66) mas, como se v e, o primeiro coeciente e c e n ao-nulo, 0 = cm , que por hip otese. Isto posto, passemos a analisar o que se passa inserindo a express ao (13.66) em (13.65). Para (13.66) valem

y (z ) = e y (z ) = a equa ca o (13.65) ca
n=0 n=0

(n + )cn z n+ 1
n=0

(13.67)

(n + )(n + 1)cn z n+ 2 ,

(13.68)

(n + )(n + 1)cn z n+ +

an z n
n=0

(n + )cn z n+ +
n=0 n=0

bn z n
n=0

cn z n+ = 0.

Usando novamente (13.46), isso ca


n=0 n n

(n + )(n + 1)cn z n+ +
n=0

anm (m + )cm
n=0 m=0

z n+ +
n=0 m=0

bnm cm

z n+ = 0.

ou seja,
n n

(n + )(n + 1)cn +

anm (m + )cm
m=0

+
m=0

bnm cm

z n+ = 0

JCABarata. Curso de F sica-Matem atica

Vers ao de 29 de janeiro de 2013.

Cap tulo 13

587/2069

que implica ( 1) + a0 + b0 c0 (n + )(n + 1) + a0 (n + ) + b0 cn para todo n 0. Como c0 = 0, temos que ( 1) + a0 + b0 (n + )(n + 1) + a0 (n + ) + b0 cn = 0,


n1

= 0,
n1

anm (m + ) + bnm cm ,
m=0

n 1 .

(13.69) anm (m + ) + bnm cm , n 1 . (13.70)

m=0

A equa ca o (13.69) e denominada na literatura equa c ao indicial, por ser uma equa ca o alg ebrica (de segundo grau) para o ndice . Antes de escrevermos a solu ca o dessa equa ca o, denotemos por f o polin omio de segundo grau f (x) = x(x 1) + a0 x + b0 = x2 + (a0 1)x + b0 . As equa co es (13.69) e (13.70) podem, claramente, ser reescritas como f ( ) = 0 ,
n1

(13.71) anm (m + ) + bnm cm , n 1 . (13.72)

f ( + n) cn

m=0

A equa ca o f ( ) = 0 e uma equa ca o alg ebrica de segundo grau, cujas solu co es s ao = 1 a0 (a0 1)2 4b0 2 e + = 1 a0 + (a0 1)2 4b0 . 2

Assim, a equa ca o indicial f ( ) = 0 obriga o ndice a ser ou + . H a dois casos a considerar: o caso + Z e o caso + Z. Trataremos primeiramente do caso + Z, que e o mais simples. Como a diferen ca + n ao e um n umero inteiro, tem-se em particular que = + . Fora isso, como e + s ao os dois u nicos zeros (distintos) do polin omio f (x), tem-se que f ( + n) = 0 para todos n 1 inteiros. Se assim n ao fosse e houvesse n0 Z com, digamos, f (+ + n0 ) = 0 valeria = + + n0 , ou seja, + = n0 , que e inteiro: uma contradi ca o. Com isso, podemos de (13.72) obter cn ( ) = 1 anm (m + ) + bnm cm ( ) f ( + n) m=0 1 2 ( + n) + (a0 1)( + n) + b0
n1 n1

O caso + Z

anm (m + ) + bnm cm ( ) ,
m=0

(13.73)

para todo n 1. Essas express oes recursivas permitem-nos obter todos os cn ( ) a partir de um c0 ( ) n ao-nulo arbitr ario e, respectivamente, todos os cn (+ ) a partir de um c0 (+ ) n ao-nulo arbitr ario. Conclu mos assim, que no caso + Z a equa ca o diferencial (13.65) (com z0 = 0) possui duas solu co es linearmente independentes y1 (z ) e y2 (z ), dadas por

y1 (z ) =
n=0

cn ( )z n+

y2 (z ) =
n=0

cn (+ )z n++ ,

JCABarata. Curso de F sica-Matem atica

Vers ao de 29 de janeiro de 2013.

Cap tulo 13

588/2069

com cn ( ) dadas por (13.73), a solu ca o geral sendo uma combina ca o linear de ambas. As constantes c0 ( ) e c0 (+ ) s ao n ao-nulas e arbitr arias. O caso + Z O caso = +

O caso + Z subdivide-se em dois: o caso + = 0 e o caso + Z \ {0}. Comecemos com o primeiro. O caso = + ocorre se e somente se (a0 1)2 4b0 = 0 e, portanto, tem-se = + = 0 , com 0 := 1 a0 . 2 (13.74)

Note-se que se (a0 1)2 4b0 = 0 a equa ca o f (x) = 0 tem apenas 0 por raiz e, portanto, f (n + 0 ) = 0 para todo n 1. Conseq uentemente, os coecientes cn com n 1 ser ao dados recursivamente por (vide (13.72)) cn (0 ) = 1 anm (m + 0 ) + bnm cm (0 ) f (0 + n) m=0 1 2 (0 + n) + (a0 1)(0 + n) + b0
n1 n1

anm (m + 0 ) + bnm cm (0 ) ,
m=0

(13.75)

para todo n 1. Como se constata, a u ltima express ao relaciona cn com os coecientes anteriores cn1 , . . . , c0 . Assim, xando apenas c0 todos os demais est ao determinados. Obtemos dessa forma, para o caso (a0 1)2 4b0 = 0 a solu ca o

y1 (z ) =
n=0

cn (0 ) z n+0 ,

(13.76)

onde os coecientes cn (0 ) s ao obtidos recursivamente de (13.75) a partir de um c0 arbitr ario. Pelo Teorema 12.4, p agina 535, a s erie acima ser a convergente (ao menos na regi ao onde as s eries de a(z ) e b(z ) convergem). Com esse proceder obtivemos apenas uma solu ca o da equa ca o diferencial (13.65). Como a mesma e de segunda ordem, uma segunda solu ca o dever a existir. Novamente, o Teorema 12.4, p agina 535, indica-nos que essa segunda solu ca o pode ter uma singularidade logar tmica. Podemos procurar essa segunda solu ca o seguindo um procedimento devido a DAlembert9 , que consiste em procurar solu co es da forma y2 (z ) = Ay1 (z ) ln(z ) + v (z ) , (13.77)

sendo y1 (z ) a solu ca o j a conhecida em (13.76) e onde A e uma constante a ser determinada, assim como a fun ca o v (z ). Note-se que o Ansatz (13.77) est a de acordo com o Teorema 12.4, p agina 535, que prev e a ocorr encia de solu co es com uma singularidade logar tmica. A especialidade do Ansatz de DAlembert est a em espertamente10 prever que o fator que multiplica ln(z ) e a primeira solu ca o y1 (z ). Substituindo (13.77) na equa ca o (13.65), obtem-se a seguinte equa ca o para v (z ):
z 2 v (z ) + za(z )v (z ) + b(z )v (z ) = A 2zy1 (z ) + (a(z ) 1)y1 (z ) .

(13.78)

E. 13.6 Exerc cio. Verique! Como facilmente se verica, o lado direito e dado pela expans ao

A
9 Jean 10 Na

fn z n+0 ,
n=0

(13.79)

Le Rond dAlembert (17171783). literatura matem atica o truque e por vezes denominado m etodo de redu c ao de DAlembert e pode ser usado em v arias equa co es diferenciais de segunda ordem para se obter uma segunda solu ca o da equa ca o a partir de uma primeira solu ca o conhecida.

JCABarata. Curso de F sica-Matem atica

Vers ao de 29 de janeiro de 2013.

Cap tulo 13

589/2069

onde fn = [2(n + 0 ) 1] cn (0 ) +

anm cm (0 ) .
m=0 n+0 . n=0 vn z

(13.80) Inserindo isso

A equa ca o (13.79) sugere que uma solu ca o para v (z ) deve ser procurada na forma v (z ) = em (13.78) tem-se
n=0 n

(n + 0 )(n + 0 1)vn +

m=0

(m + 0 )anm + bnm vm z n+0 = A


n

fn z n+0 ,
n=0

que implica (n + 0 )(n + 0 1)vn + para todo n 0. Para n = 0 a rela ca o acima e

m=0

(m + 0 )anm + bnm vm = Afn

0 (0 1) + a0 0 + b0 v0 = Af0 , que e uma identidade trivial, j a que 0 (0 1) + a0 0 + b0 = 0 e que f0 = 0 [20 1 + a0 ] c0 (0 ) = 0, por (13.74). Para n 1 tem-se, por em, vn = 1 (0 + n)2 + (0 + n)(a0 1) + b0
n1

Afn +
m=0

(m + 0 )anm + bnm vm

n 1 ,

(13.81)

o que permite obter recursivamente todos os vn a partir de v0 . Observemos agora que A deve, nesse caso, ser for cosamente n ao-nulo, pois se tom assemos A = 0 ter amos por (13.81) e (13.75) que os coecientes vn satisfazem as mesmas rela co es de recorr encia dos cn (0 ). Assim, v (z ) e y1 (z ) n ao seriam linearmente independentes. Podemos, portanto, adotar sem perda de generalidade, A = 1. Com essa escolha e expressando-se os fn s como em (13.80), tem-se
n

vn (0 ) =

1 2 (0 + n) + (0 + n)(a0 1) + b0

[2(n + 0 ) 1] cn (0 ) +
n1

anm cm (0 )
m=0

+
m=0

(m + 0 )anm + bnm vm ,

n 1 , (13.82)

que expressa os vn s em termos dos coecientes cn (0 ) de y1 (z ), os quais, por sua vez, s ao dados pelas rela co es recursivas (13.75)11 , e de v0 (0 ) arbitr ario. Resumindo nossas conclus oes, caso (a0 1)2 4b0 = 0, a solu ca o da equa ca o diferencial (13.65) (com z0 = 0) possui duas solu co es linearmente independentes y1 (z ) e y2 (z ), dadas por

y1 (z ) =
n=0

cn (0 )z n+0

y2 (z ) = y1 (z ) ln(z ) +
n=0

vn (0 )z n+0 ,

com 0 = (1 a0 )/2, com os cn (0 )s dados em (13.75) e com os vn (0 )s dados em (13.82), tomando-se A = 1. As constantes c0 ( ) e v0 ( ) s ao n ao-nulas e arbitr arias. E de se notar que, como A e n ao-nulo, uma das solu co es possui uma singularidade logar tmica. O caso + Z \ {0}

Esse u ltimo caso, com a generalidade com que o abordamos aqui, e o mais complexo e o estudante poder a dispensar seu estudo detalhado em uma primeira leitura, atendo-se preferencialmente aos exemplos das equa co es de Bessel e Laguerre, das quais trataremos adiante.
11 Vide

nota de rodap e da p agina 583.

JCABarata. Curso de F sica-Matem atica

Vers ao de 29 de janeiro de 2013.

Cap tulo 13

590/2069

O caso + Z \ {0} e semelhante ao caso anterior onde = + , a principal diferen ca sendo que aqui podem ocorrer situa co es onde A = 0, de modo que ambas as solu co es podem ser livres de singularidades logar tmicas. De fato, sabe-se de equa co es particulares onde tem-se A = 0 (um exemplo sendo a equa ca o de Bessel de ordem 1/2) e de equa co es particulares onde tem-se A = 0 (um exemplo sendo a equa ca o de Bessel de ordem 1). Comecemos com algumas deni co es. O caso + Z \ {0} s o pode ocorrer se n ao-nulo. Denamos ent ao n0 = (a0 1)2 4b0 . (a0 1)2 4b0 for um inteiro

Claro est a que n0 {1, 2, 3, 4, . . .}. Como + e um inteiro n ao-nulo, denamos tamb em 1 := , 2 := + , caso + 1, ou (13.83) 1 := + , 2 := , caso + 1.

Com essas deni co es, est a sempre garantido que 1 = 2 + n 0 . Isso diz-nos que para todo n 1 a express ao f (1 + n) n ao pode se anular, pois se assim o fosse ter amos for cosamente 1 + n = 2 , ou seja, n = n0 , um absurdo, j a que n0 1. Por outro lado, existe um u nico valor de n para o qual f (2 + n) se anula, a saber n = n0 . Com isso em mente, vemos que para a solu ca o = 1 da equa ca o indicial, a express ao (13.72) permite-nos obter todos os coecientes cn a partir de um c0 n ao-nulo: cn (1 ) = 1 anm (m + 1 ) + bnm cm (1 ) f (1 + n) m=0 1 (1 + n)2 + (a0 1)(1 + n) + b0
n1 n1

anm (m + 1 ) + bnm cm (1 ) ,
m=0

(13.84)

para todo n 1. Isso fornece-nos a primeira solu ca o da equa ca o diferencial (13.65) (com z0 = 0):

y1 (z ) =
n=0

cn (1 )z n+1 ,

(13.85)

com os cn (1 ) dados em (13.84) em termos de c0 (1 ), arbitr ario mas n ao-nulo. Passemos a procurar a segunda solu ca o independente da equa ca o diferencial (13.65). O caso da solu ca o = 2 da equa ca o indicial requer cuidado pois, como comentamos, vale que f (2 + n0 ) = 0. Assim, para n = n0 a equa ca o (13.72) s o faz sentido se o lado direito for igualmente nulo:
n0 1

an0 m (m + 2 ) + bn0 m cm (2 ) = 0 .
m=0

(13.86)

Essa rela ca o pode ou n ao ser satisfeita, dependendo da equa ca o diferencial tratada. Por exemplo, no caso da equa ca o de Bessel de ordem semi-inteira (ou seja, de ordem 1/2, 3/2, 5/2 etc.) verica-se que a rela ca o (13.86) e satisfeita. J a no caso da equa ca o de Bessel de ordem inteira verica-se que a rela ca o (13.86) n ao e satisfeita. Isso ser a discutido explicitamente na Se ca o 13.2.3, p agina 595. Devemos, portanto, separar provisoriamente os dois casos: aquele no qual (13.86) e satisfeita e aquele no qual n ao e. Posteriormente veremos que essa separa ca o e sup erua, mas por ora ela e logicamente necess aria. Na situa ca o feliz em que (13.86) e satisfeita, o coeciente cn0 (2 ) ca indeterminado e pode ser escolhido livremente, j a que as equa co es recursivas (13.72) n ao o xam e nada mais h a para x a-los. Com isso, as equa co es recursivas (13.72)

JCABarata. Curso de F sica-Matem atica

Vers ao de 29 de janeiro de 2013.

Cap tulo 13

591/2069

determinam todos os demais coecientes cn (2 ), n 1, n = n0 , a partir de um c0 (2 ) n ao-nulo mas arbitr ario. Assim, obtemos a solu ca o

y2 (z ) =
n=0

cn (2 )z n+2 ,

(13.87)

com cn (2 ) = 1 anm (m + 2 ) + bnm cm (2 ) f (2 + n) m=0 1 (2 + n)2 + (a0 1)(2 + n) + b0


n1 n1

anm (m + 2 ) + bnm cm (2 ) ,
m=0

(13.88)

aria12. para todo n 1, n = n0 e cn0 (2 ) = constante arbitr

Resta-nos ainda tratar do caso em que a rela ca o (13.86) n ao e satisfeita. Aqui, devemos proceder como zemos no caso = + e procurar uma solu ca o na forma y2 (z ) = Ay1 (z ) ln(z ) + v (z ), com A sendo uma constante e y1 sendo a solu ca o j a conhecida (13.85). Substituindo isso na equa ca o (13.65), obtem-se novamente a equa ca o (13.78) para v (z ). Como facilmente se verica, o lado direito de (13.78) e dado pela expans ao

A onde, como antes,

n=0

gn (1 )z n+1 = A

gn (1 )z n+n0 +2 ,
n=0

(13.89)

gn (1 ) = [2(n + 1 ) 1] cn (1 ) + os coecientes cm (1 ) sendo dados por (13.84).

anm cm (1 ) ,
m=0

n0,

(13.90)

A equa ca o (13.89) sugere que uma solu ca o para v (z ) deve ser procurada na forma

v (z ) =
n=0

vn z n+2 .

Inserindo isso em (13.78) tem-se


n=0 n n

(n + 2 )(n + 2 1)vn +

anm (m + 2 )vm
m=0

+
m=0

bnm vm

z n+2 = A

gnn0 (1 )z n+2 ,
n=n0

o que implica
n

(n + 2 )(n + 2 1)vn +

(m + 2 )anm + bnm vm
m=0

0,

n = 0 , . . . , n0 1 ,

(13.91)

(n + 2 )(n + 2 1)vn +

(m + 2 )anm + bnm vm
m=0

Agnn0 (1 ),

n n0 .

(13.92)

e f (2 ) = 0, Para n = 0 a rela ca o (13.91) tem a forma 2 (2 1) + a0 2 + b0 v0 = 0, mas como o fator entre colchetes conclu mos que essa rela ca o e trivialmente satisfeita e, assim, v0 pode ser escolhido livremente. Para 1 n n0 1,
12 O que ocorre se, por op ao-nulo? Nesse caso ter amos um termo a mais em y2 (z ) do tipo cn0 z n0 +2 = cn0 z 1 . ca o, escolhermos cn0 (2 ) n Esse termo se adicionaria na solu ca o geral ao termo c0 (1 )z 1 proveniente da solu ca o y1 (z ), ou seja, corresponderia a uma nova escolha da constante arbitr aria c0 (1 ), n ao representando, assim, nenhuma mudan ca na solu ca o geral.

JCABarata. Curso de F sica-Matem atica

Vers ao de 29 de janeiro de 2013.

Cap tulo 13

592/2069

(13.91) implica que vn 1 = (m + 2 )anm + bnm vm f (2 + n) m=0 = 1 (2 + n)2 + (a0 1)(2 + n) + b0


n1 n1

(13.93)

(m + 2 )anm + bnm vm
m=0

(13.94)

Para n = n0 a rela ca o (13.92) e


n0 1

(n0 + 2 )(n0 + 2 1) + a0 (n0 + 2 ) + b0 vn0 +

m=0

(m + 2 )an0 m + bn0 m vm = A[21 1 + a0 ] c0 (1 ) .

Como (n0 + 2 )(n0 + 2 1) + a0 (n0 + 2 ) + b0 = f (n0 + 2 ) = f (1 ) = 0, camos apenas com


n0 1 m=0

[(m + 2 )an0 m + bn0 m ] vm = A[21 1 + a0 ] c0 (1 ) = A

(a1 1)2 4b0 c0 (1 ) ,

(13.95)

f o sinal dependendo de se ter 1 = + ou 1 = , respectivamente. E acil ver, por em, que em qualquer caso 2 ca o (13.95) xa A: (a1 1) 4b0 = n0 . A rela A = 1 c0 (1 ) n0
n0 1

[(m + 2 )an0 m + bn0 m ] vm ,


m=0

(13.96)

com os vm xados na express ao (13.94) em fun ca o de v0 = 0 arbitr ario. ao e xado por nenhuma das rela co es anteriores e pode ser escolhido livremente. Sua presen ca O coeciente vn0 n a solu ca o geral e aplica-se novamente o coment ario de rodap e da p agina adiciona um termo do tipo vn0 z n0 +2 = vn0 z 1 ` 591. Para n > n0 , tem-se ainda por (13.92) vn = 1 f (2 + n)
n1

Agnn0 (1 ) +
m=0

anm (m + 2 ) + bnm vm

1 (2 + n)2 + (2 + n)(a0 1) + b0

n1

Agnn0 (1 ) +
m=0

anm (m + 2 ) + bnm vm

. (13.97)

com os gn (1 ) xados em (13.90) em termos dos coecientes cm (1 ) da solu ca o y1 (z ). As express oes (13.94), (13.96) e (13.97) permitem xar todos os vn s e a constante A em termos de v0 = 0 e de vn0 , arbitr arios. Observemos, A n ao e for cosamente nulo, nem pode ser escolhido arbitrariamente. Sobre a constante A vale ainda uma observa ca o importante. A condi c ao (13.86) e a constante A

Observe o leitor que as rela co es de recorr encia (13.94), que xam os vm s com m = 0, . . . , n0 1, s ao id enticas ` as de (13.88), que xam todos os cm (2 )s, em particular aqueles com m = 0, . . . , n0 1. Os vm s s ao xados por um v0 inicial n ao-nulo e os cm (2 )s por um c0 (2 ) inicial n ao-nulo. Contemplando aquelas rela co es de recorr encia, um minuto de medita ca o nos leva a perceber que todos os vm s ao proporcionais a v0 e que todos os cm (2 ) s ao proporcionais a c0 (2 ). Como as rela co es de recorr encia s ao id enticas, conclu mos que vm = v0 cm (2 ) c0 (2 ) para todo m = 0, . . . , n0 1 .

JCABarata. Curso de F sica-Matem atica

Vers ao de 29 de janeiro de 2013.

Cap tulo 13

593/2069

Agora, pela express ao (13.96), A e proporcional a


n0 1

[(m + 2 )an0 m + bn0 m ] vm =


m=0

v0 c0 (2 )

n0 1

[(m + 2 )an0 m + bn0 m ] cm (2 ) .


m=0

Au ltima soma, por em, e id entica ` aquela de (13.86)! Assim, percebemos que, sob a hip otese que (13.86) n ao e satisfeita, tem-se que A = 0. Por outro lado, se (13.86) e satisfeita, ent ao A = 0. Mas se A = 0, as rela co es de recorr encia (13.97) tornam-se tamb em id enticas ` aquelas de (13.88), que xam todos os cm (2 )s. Conclu mos ent ao, que nesse caso em que A = 0 (ou seja, sob (13.65)) vale tamb em v0 vm = cm (2 ) , c0 (2 ) mas agora para todo m 0. Assim, para A = 0 a solu ca o y2 (z ) = A ln(z )y1 (z )+ v (z ) reduz-se (a menos de uma constante multiplicativa trivial) ` a solu ca o para y2 (z ) dada em (13.87), obtida sob a condi ca o (13.86). Nesse sentido, a condi ca o (13.86) e sup erua e podemos unicar as solu co es que obtivemos nos casos em que (13.86) e ou n ao e satisfeita e resumir nossas conclus oes da seguinte forma: Para + Z \ {0}, a equa ca o diferencial (13.65) (com z0 = 0) tem duas solu co es independentes y1 (z ) e y2 (z ), onde: y1 (z ) =
n=0

cn (1 )z n+1

y2 (z ) = Ay1 (z ) ln(z ) +
n=0

vn z n+2 ,

onde os cn (1 ), n 1, tamb em est ao denidos em (13.84) a partir de um c0 (1 ) n ao-nulo arbitr ario e onde os vn s com arios. n 1, n = n0 , e a constante A s ao xados em (13.94), (13.96) e (13.97) em termos de v0 = 0 e de vn0 , arbitr

Como mencionamos, h a casos em que A = 0, exemplos sendo as equa ca o de Bessel de ordem semi-inteira e a equa ca o de Euler, para certos par ametros. Com tudo isso a demonstra ca o do Teorema 13.2 est a completa e podemos passar ao estudo de exemplos particulares.

13.2.2

A Equa c ao de Euler Revisitada


z 2 y (z ) + azy (z ) + by (z ) = 0 ,

A equa c ao de Euler13 (de segunda ordem) e a equa ca o diferencial

onde a e b s ao constantes. Comparando com a forma (13.52), vemos vemos que a(z ) = a e que b(z ) = b. Assim, no presente caso tem-se a, para n = 0 , bn = an = 0, para n 1

que z0 = 0 e um ponto singular regular da equa ca o, b, 0,

para n = 0 para n 1

A equa ca o de Euler j a foi resolvida ` a p agina 535, onde encontramos as solu co es (12.83) e (12.84). Vamos trat a-la aqui sob a luz do Teorema 13.2, p agina 583. Se procurarmos uma solu ca o na forma y (z ) =
n=0

cn z n+ ,

(13.98)

com y (z ) =

(n + )cn z n+ 1
n=0

(13.99)

13 Leonhard Euler (17071783). Um dos matem aticos mais prol cos e inuentes de todos os tempos, Euler foi um dos fundadores da teoria das equa co es diferenciais e deixou contribui co es seminais em in umeros campos da Matem atica e da F sica. A equa ca o de Euler apresentada abaixo e uma das v arias que levam seu nome. H a uma outra equa ca o de Euler na Mec anica dos Fluidos, assim como f ormulas de Euler, invariantes de Euler, m etodos de Euler, Ans atze de Euler, multiplicadores de Euler, constantes de Euler, n umeros de Euler, angulos de Euler, problemas de Euler, conjecturas de Euler, teoremas de Euler etc. Boa parte da nota ca o matem atica usada atualmente e tamb em sua inven ca o (por exemplo, o s mbolo f para denotar a derivada de uma fun ca o f ou o uso da letra e para designar o n umero 2, 7182818 . . .).

JCABarata. Curso de F sica-Matem atica

Vers ao de 29 de janeiro de 2013.

Cap tulo 13

594/2069

e y (z ) = a equa ca o de Euler ca
n=0

n=0

(n + )(n + 1)cn z n+ 2 ,

(13.100)

(n + )(n + 1)cn z
n=0

n+

+
n=0

a(n + )cn z

n+

+
n=0

bcn z n+ = 0

ou seja,

(n + )(n + 1)cn + a(n + )cn + bcn z n+ = 0, f (n + ) cn = 0 n 0.

o que implica onde f e o polin omio de segundo grau. f (x) := x(x 1) + ax + b = x2 + (a 1)x + b . Sem perda de generalidade, podemos sempre adotar c0 = 0, pois se cm fosse o primeiro coeciente n ao-nulo, a s erie n+ := n+ c e = + m , que tem a mesma forma gen e rica c z com c c z poderia ser reescrita como n + m n n n=0 n n=0 mas com c 0 = 0. Assim, devemos impor f ( ) = 0, o que possui duas solu co es: = 1a (a 1)2 4b 2 e + = 1a+ (a 1)2 4b . 2

Se + n ao for um inteiro, a equa ca o f ( + n) = 0 n ao e satisfeita para nenhum n 1 inteiro. A raz ao ea seguinte: f e um polin omio de segundo grau e, portanto, possui apenas duas solu co es. Assim, se f ( + n) = 0 ter amos + n = , o que implica que + e inteiro, uma contradi ca o. Nesse caso, ent ao, temos que adotar cn = 0 para todo n 1 e as solu co es da equa ca o de Euler cam y1 (z ) = z e y2 (z ) = z + . (13.101)

No caso de = + = 0 = (1 a)/2, tem-se por (13.56) uma solu ca o na forma


y1 (z ) = z 0
n=0

cn (0 )z n

e uma segunda na forma

y2 (z ) = y1 (z ) ln(z ) + z 0
n=0

vn (0 )z n ,

com os cn dados em (13.57) e os vn dados em (13.58). Observando (13.57), constata-se que nesse caso cn (0 ) = 0 para todo n, exceto n = 0, pois apenas a0 e b0 podem ser n ao-nulos. Igualmente, observando (13.58) constata-se que vn (0 ) e proporcional a cn (0 ) para todo n 1 e, com isso, apenas v0 pode ser n ao-nulo. Assim, temos nesse caso, tomando c0 = v0 = 1, e y2 (z ) = z 0 ln(z ) + z 0 . y1 (z ) = z 0 ao de y2 (z ) e o pr oprio y1 (z ), de modo que podemos tomar como solu co es linearmente indepenO termo z 0 na express dentes as seguintes: (13.102) e y2 (z ) = z 0 ln(z ) . y1 (z ) = z 0 Por m, consideremos o caso em que + e um inteiro n ao-nulo. Denamos 1 e 2 como em (13.59), com n0 = | (a 1)2 4b|.
n=0

a a forma y2 (z ) = Ay1 (z ) ln(z ) + z 2 Ent ao uma solu ca o ser a y1 (z ) = z 1 n=0 cn (1 )z n e a outra ter aqui os cn s ao dados em (13.61), os vn s ao dados em (13.62) e A e dada em (13.63).

vn z n onde

JCABarata. Curso de F sica-Matem atica

Vers ao de 29 de janeiro de 2013.

Cap tulo 13

595/2069

Contemplando (13.61) constata-se que cn (1 ) = 0 para todo n 1, pois apenas a0 e b0 podem ser n ao-nulos, sendo que podemos escolher c0 = 1, livremente. Disso conclu mos que y1 (z ) = z 1 . Por (13.63) tem-se que A = 0 pois, no caso da equa ca o de Euler, an0 m = bn0 m = 0 para m = 0, . . . , n0 1. Por (13.62), tem-se analogamente 0, para 1 n n0 1 , vn = arbitr ario , para n = n0 , 0, para n > n0 , ao arbitr arios, sendo que v0 deve ser n ao-nulo. Escolhendo v0 = 1 e vn0 = 0, segue que Assim, apenas v0 e vn0 s y2 (z ) = z 2 . Concluindo, vale aqui que y1 (z ) = z 1 e y2 (z ) = z 2 . (13.103)

Todos esses resultados coincidem, como deveria ser, com aqueles obtidos em (12.83) e (12.84), p agina 535 e seguintes. O estudo das solu co es da equa co es de Euler eu til na resolu ca o de equa co es com singularidades regulares mais gerais como z 2 y (z ) + za(z )y (z ) + b(z )y (z ) = 0 pela seguinte raz ao. Pr oximo ao ponto singular z0 = 0, podemos aproximar a(z ) a0 e b(z ) b0 , j a que esses s ao os primeiros termos das expans oes de Taylor de a(z ) e b(z ). Assim, para |z | pequeno o suciente, a equa ca o aproxima-se de z 2 y (z ) + a0 z y (z ) + b0 y (z ) = 0 que e uma equa ca o de Euler com a = a0 e b = b0 . Com isso, vemos que as solu co es da equa ca o geral se aproximam para |z | pequeno daquelas encontradas em (13.101), (13.102) ou (13.103), dependendo do caso. Esse proceder permite-nos, face a uma equa ca o singular regular geral, estudar qual tipo de singularidade deve ocorrer pr oximo ao ponto singular e, com isso, perceber qual das solu co es descritas no Teorema 13.2, p agina 583, se aplica. Em verdade, a resolu ca o da equa ca o indicial (13.53) fornece o mesmo tipo de informa ca o.

13.2.3

A Equa c ao de Bessel

Uma das equa co es diferenciais mais importantes dentro da classe que temos estudado e a equa ca o de Bessel, a qual surge em v arios problemas de F sica e de Matem atica aplicada. A mesma pode ser encontrada, por exemplo, quando da resolu ca o da equa ca o de Helmholtz em duas dimens oes em coordenadas polares ou em tr es dimens oes em coordenadas esf ericas (levando ` as chamadas fun c oes de Bessel esf ericas). Vide para tal o Cap tulo 19, p agina 842. Para alguns coment arios hist oricos sobre a origem das equa co es de Bessel e das fun co es de Bessel, vide p agina 672. A equa ca o diferencial com z C, onde C e uma constante, e denominada equa c ao de Bessel14 de ordem . Comparando com a forma (13.52), vemos que z0 = 0 e um ponto singular regular da equa ca o, vemos que a(z ) = 1 e que b(z ) = z 2 2 . Assim, no presente caso tem-se 2 , para n = 0 1, para n = 0 . , bn = an = 1, para n = 2 0, para n 1 0, para n = 1 ou n 3 A equa ca o indicial (13.53) conduz ` as solu co es =
14 Friedrich

z 2 y (z ) + zy (z ) + (z 2 2 )y (z ) = 0 ,

(13.104)

+ = .

Wilhelm Bessel (17841846).

JCABarata. Curso de F sica-Matem atica

Vers ao de 29 de janeiro de 2013.

Cap tulo 13

596/2069

H a, portanto, tr es casos a considerar: 1. o caso em que 2 Z, 2. o caso em que 2 = 0 e 3. o caso em que 2 Z \ {0}. Observe o leitor que as condi co es 2 e 3 correspondem a semi-inteiro ou inteiro. Os dois casos s ao os mais relevantes em F sica. O caso de inteiro conduz ` as chamadas fun c oes de Bessel e o caso de semi-inteiro conduz ` as chamadas fun c oes de Bessel esf ericas as quais surgem, por exemplo, em problemas de propaga ca o de ondas em duas ou tr es dimens oes, respectivamente. Vide Se ca o 13.2.4, p agina 605. Para a origem das fun co es de Bessel, vide nota hist orica a p ` agina 672. Nesse caso tem-se duas solu co es y =
n=0

Caso 1. 2 Z.

cn ( )z n ,

com cn ( ) dados por (13.55): cn ( ) = 1 n(n + 2 )


n1 m=0

(m )anm + bnm cm ( ) .

Podemos nos concentrar apenas nos coecientes cn (+ ), pois os coecientes cn ( ) podem ser obtidos fazendo-se . Vale n1 1 cn ( ) = (13.105) (m + )anm + bnm cm ( ) , n(n + 2 ) m=0 e tem-se c1 ( ) c2 ( ) = 0, = = 1 c0 ( ) , 2(2 + 2 ) 1 cn2 ( ), n(n + 2 ) n 3.

cn ( ) Com isso, ca claro que c2k ( ) c2k+1 ( ) = =

(1)k c0 ( ) , (2k )!! (2 + 2 )(4 + 2 ) (2k + 2 ) 0, k0.

k0.

E. 13.7 Exerc cio importante. Mostre isso! Au ltima express ao pode ser reescrita como c2k ( ) c2k+1 ( ) = = (1)k c0 ( ) , k ! 22k (1 + )(2 + ) (k + ) 0, k0, k0.

onde usamos que (2 + 2 )(4 + 2 ) (2k + 2 ) = 2k (1 + )(2 + ) (k + ) e tamb em que (2k )!! = 2k k !. Como a fun ca o denida em (13.25)-(13.26) satisfaz (k + 1 + ) = (1 + )(1 + )(2 + ) (k + ) , podemos ainda escrever c2k ( ) c2k+1 ( ) = = (1)k (1 + ) c0 ( ) , k ! 22k (k + 1 + ) 0, k0. k0.

JCABarata. Curso de F sica-Matem atica

Vers ao de 29 de janeiro de 2013.

Cap tulo 13

597/2069

Por conven ca o hist orica adota-se c0 ( ) = e chega-se com isso ` a express ao J (z ) :=


k=0

1 (1 + ) z 2
2k+

(1)k k ! (k + 1 + )

(13.106)

Essa fun ca o representa uma das solu co es da equa ca o de Bessel de ordem para o caso considerado e e denominada fun c ao de Bessel de primeiro tipo e ordem . Como comentamos, uma segunda solu ca o e obtida fazendo-se :

J (z ) :=
k=0

(1)k k ! (k + 1 )

z 2

2k

Conclu mos, assim, com a constata ca o que a solu ca o geral da equa ca o de Bessel de ordem para o caso 2 Z e 1 J (z ) + 2 J (z ) , onde 1 e 2 s ao constantes arbitr arias. Por conven ca o hist orica, e costume considerar-se tamb em uma combina ca o linear particular de J (z ), a saber a seguinte: J (z ) cos( ) J (z ) N (z ) := . (13.107) sen ( ) Essa fun ca o N (z ) tamb em representa uma das solu co es da equa ca o de Bessel de ordem (por ser uma combina ca o linear de duas outras) e e denominada fun c ao de Bessel de segundo tipo e ordem , ou ainda fun c ao de Neumann15 de ordem . Conclu mos, assim, que a solu ca o geral da equa ca o de Bessel de ordem para o caso 2 Z tamb em pode ser escrita em termos das fun co es J e N na forma 1 J (z ) + 2 N (z ) , onde 1 e 2 s ao constantes arbitr arias. O estudante deve notar que as fun co es J (z ) e N (z ), para 2 n ao-inteiro, s ao anal ticas em toda a parte, exceto em z = 0, onde possuem um ponto de ramica ca o devido ao fator z = exp( ln(z )). Caso 2. 2 = 0. No caso em quest ao aplicam-se as solu co es (13.56), (13.57) e (13.58). Aqui tem-se 0 = (1 a0 )/2 = 0 e para y1 tem-se y1 (z ) = n=0 cn (0)z n , com (por (13.57)) cn (0) = 1 n2
n1

manm + bnm cm (0) .


m=0

Essas rela co es s ao id enticas ` aquelas de (13.105) (tomando-se aqui = 0) e, assim, tem por solu ca o c2k (0) c2k+1 (0) = (1)k (1)k (1) c0 (0) , c (0) , = 0 k ! 22k (k + 1) (k !)2 22k k0

k0,

= 0,

onde usamos que (1) = 1 e (k + 1) = k !. Por conven ca o hist orica adota-se c0 (0) = 1 e chega-se com isso ` a express ao J0 (z ) =
k=0

(1)k (k !)2

z 2

2k

(13.108)

Essa fun ca o representa uma das solu co es da equa ca o de Bessel de ordem 0 e e denominada fun c ao de Bessel de primeiro tipo e ordem 0.
15 Carl

Neumann (18321925).

JCABarata. Curso de F sica-Matem atica

Vers ao de 29 de janeiro de 2013.

Cap tulo 13

598/2069

Para a segunda solu ca o y2 teremos, por (13.56),

y2 (z ) = J0 (z ) ln(z ) +
n=0

vn z n ,

com os vn dados em (13.58). Como o estudante pode facilmente vericar, adotando-se v0 = 0, obtem-se para esses coecientes as seguintes express oes: v2k v2k+1 onde h0 hn Note-se que v0 = 0. E. 13.8 Exerc cio importante. Verique! Com isso, a segunda solu ca o y2 (z ) ser a

= =

(1)k+1 hk , (k !)2 22k 0, k0

k0,

:= :=

0, 1+ 1 1 1 + + + = 2 3 n
n

(13.109) 1 , l n1. (13.110)

l=1

y2 (z ) = J0 (z ) ln(z ) +
k=1

(1)k+1 hk (k !)2

z 2

2k

(13.111)

Por conven ca o hist orica, costuma-se considerar tamb em uma particular combina ca o das solu co es J0 (z ) e y2 (z ): N0 (z ) := 2 y2 (z ) + ( ln(2))J0 (z ) = 2 + ln z 2

J0 (z ) +
k=1

(1)k+1 hk (k !)2

z 2

2k

(13.112)

onde e a chamada constante de Euler-Mascheroni16 , denida por17 : := lim (hn ln(n)) = lim
n

1+

1 1 1 + + + ln(n) 2 3 n

0, 5772156649 . . . .

Essa fun ca o N0 (z ) tamb em representa uma das solu co es da equa ca o de Bessel de ordem 0 (por ser uma combina ca o linear de duas outras) e e denominada fun c ao de Bessel de segundo tipo e ordem 0, ou ainda fun c ao de Neumann de ordem 0. Conclu mos, assim, com a constata ca o que a solu ca o geral da equa ca o de Bessel de ordem 0 e 1 J0 (z ) + 2 N0 (z ) , onde 1 e 2 s ao constantes arbitr arias. O estudante deve notar que a primeira solu ca o J0 (z ) e uma fun ca o anal tica para todo z C (pois a s erie em (13.108) converge absolutamente para todo z (mostre isso!)). J a a solu ca o N0 (z ) e tamb em anal tica em toda parte, exceto em z = 0, onde possui uma singularidade logar tmica. Caso 3. 2 Z \ {0}.
Euler (17071783). Lorenzo Mascheroni (17501800). constante foi introduzida por Euler em 1735, o qual calculou seus 16 primeiros d gitos decimais. Em 1790, Mascheroni calculou seus 32 primeiros d gitos decimais, dos quais apenas os primeiros 19 estavam corretos.
17 Essa 16 Leonhard

JCABarata. Curso de F sica-Matem atica

Vers ao de 29 de janeiro de 2013.

Cap tulo 13

599/2069

Como a equa ca o de Bessel e invariante por , podemos sem perda de generalidade tomar aqui 2 um inteiro positivo. Como veremos, h a dois casos a considerar: a. e um inteiro positivo e b. e um semi-inteiro positivo, ou seja, no caso a. tem-se = 1, 2, 3, 4, . . . enquanto que no caso b. tem-se = 1/2, 3/2, 5/2, . . .. Caso a. = 1, 2, 3, 4, . . .. Vamos aqui escrever = p, com p sendo um inteiro positivo: p = 1, 2, 3, 4, . . .. Com essas conven co es, tem-se que 1 = p, 2 = p e n0 = 2p. As solu co es y1 e y2 s ao aquelas dadas em (13.60), (13.61) e (13.62):

y1 (z ) = z p
n=0

cn (p)z n

y2 (z ) = Ay1 (z ) ln(z ) + z p
n=0

vn z n ,

onde, segundo (13.61), as constantes cn (p) satisfazem cn (p) = 1 (m + p)anm + bnm cm (p) f (p + n) m=0
n1

para n 1. Novamente, essas rela co es s ao id enticas ` aquelas de (13.105) e, assim, suas solu co es s ao c2k (p) c2k+1 (p) = (1)k p! (1)k (1 + p) c ( p ) = c0 (p) , 0 k ! 22k (k + 1 + p) k ! 22k (k + p)! k0,
1 2p p!

k0.

= 0,

onde usamos que (1 + p) = p! e (k + 1 + p) = (k + p)!. Por conven ca o hist orica adota-se c0 (p) = isso ` a express ao z 2k+p (1)k . Jp (z ) = k ! (k + p)! 2
k=0

e chega-se com

Essa fun ca o representa uma das solu co es da equa ca o de Bessel de ordem p (com p = 1, 2, 3, 4, . . .) e e denominada fun c ao de Bessel de primeiro tipo e ordem p. O leitor e convidado a constatar que a express ao (13.108) para J0 (z ) e id entica a essa se tomarmos p = 0. Na Figura 13.2, p agina 600, exibimos o gr aco de algumas das primeiras fun co es de Bessel de ordem inteira. Procuremos agora a segunda solu ca o y2 (z ):

y2 (z ) = AJp (z ) ln(z ) + z p
n=0

vn (p)z n .

Por (13.62), n1 1 (m p)anm + bnm vm (p) , f (n p) m=0 vn (p) = arbitr ario , n1 1 (m p)anm + bnm vm (p) f (n p) Agn2p + m=0 1 2p c0 (p)
2p1

para 1 n 2p 1 ,

para n = 2p ,

(13.113)

para n > 2p,

A constante A e dada em (13.63) e, para o presente caso, tem-se A =

m=0

[(m p)a2pm + b2pm ] vm (p) =

2 p p! v2p2 (p) . 2p

JCABarata. Curso de F sica-Matem atica

Vers ao de 29 de janeiro de 2013.

Cap tulo 13

600/2069

1.0

J0

J1
0.5

J2

J3

0.0 0 2 4 6 8 10 12 14 16 18

20 x

0.5

Figura 13.2: Gr acos das fun co es de Bessel Jm (x), m = 0, . . . , 3, para x [0, 20].

Agora, por (13.62), v2p2 (p) = de onde se v e imediatamente que v2p2 (p) = e, portanto, v2p2 (p) = 1 f (p 2)
2p3

m=0

(m p)a2p2m + b2p2m vm (p) ,

1 v2p4 (p), 22 (p 1) 1 v0 (p), 22(p1) (p 1)!

p2, p2.

Logo, A = 4v0 (p). Adotando-se v0 (p) = 1/4 teremos A = 1 e

y2 (z ) = Jp (z ) ln(z ) + z p
n=0

vn (p)z n .

com 1 f ( n p)
n1 m=0

(m p)anm + bnm vm (p) ,

para 1 n 2p 1 ,

vn (p) =

arbitr ario , n1 1 (m p)anm + bnm vm (p) g + n2p f (n p) m=0

para n = 2p ,

(13.114)

para n > 2p,

JCABarata. Curso de F sica-Matem atica

Vers ao de 29 de janeiro de 2013.

Cap tulo 13

601/2069

com os gn dados em (13.64) em termos de cn (p). Um c alculo trabalhoso, que nos poupamos de apresentar em detalhe, conduz ao seguinte resultado: y2 (z ) = Jp (z ) ln(z ) com p = 1, 2, 3, 4, . . .. E. 13.9 Exerc cio. Tome uma hora livre e mostre isso. O leitor e convidado a constatar que a express ao (13.111) e id entica a essa se tomarmos p = 0 (com a conven ca o que = 0). Por conven ca o hist orica, costuma-se considerar tamb em uma particular combina ca o das solu co es Jp (z ) e y2 (z ): 2 y2 (z ) + ( ln(2))Jp (z ) 2 + ln z 2 Jp (z ) 1 (p n 1)! z 2 n=0 n! 2
p1 2np

(1)n (hn + hn+p ) 1 2 n=0 n! (n + p)!

z 2

2n+p

1 n=0 ( )

Np (z ) :=

= 1 (p n 1)! 2 n=0 n!
p1

z 2

2np

(1)n (hn + hn+p ) 1 2 n=0 n! (n + p)!

z 2

2n+p

, (13.115)

onde e a constante de Euler-Mascheroni mencionada acima. Essa fun ca o Np (z ) tamb em representa uma das solu co es da equa ca o de Bessel de ordem p (por ser uma combina ca o linear de duas outras) e e denominada fun c ao de Bessel de segundo tipo e ordem p, ou ainda fun c ao de Neumann de ordem p. Na Figura 13.3, p agina 601, s ao exibidos gr acos de algumas das primeiras fun co es de Neumann.
1 0 0 1 N1 2 3 4 5 6 7 8 9 5 10 15

N0

N2

Figura 13.3: Gr acos das fun co es de Neumann Nm (x), m = 0, . . . , 2, para x [1/2, 15]. Todas divergem em x = 0, a diverg encia sendo tanto mais forte quanto maior m.

Conclu mos, assim, com a constata ca o que a solu ca o geral da equa ca o de Bessel de ordem p, p = 1, 2, 3, 4, . . ., e 1 Jp (z ) + 2 Np (z ) , onde 1 e 2 s ao constantes arbitr arias.

JCABarata. Curso de F sica-Matem atica

Vers ao de 29 de janeiro de 2013.

Cap tulo 13

602/2069

O estudante deve notar que a primeira solu ca o Jp (z ) e uma fun ca o anal tica para todo z C (pois a s erie em (13.108) converge absolutamente para todo z (mostre isso!)). J a a solu ca o Np (z ) e tamb em anal tica em toda parte, exceto em z = 0, onde possui uma singularidade logar tmica assim como um polo de ordem p. Advert encia. As fun co es de Neumann s ao tamb em por vezes denotadas por Y . Precisamos estudar ainda o caso em que e um n umero semi-inteiro onde, diferentemente do caso que acabamos de estudar, as solu co es independentes s ao ambas livres de singularidades logar tmicas. Caso b. = 1/2, 3/2, 5/2, . . .. Vamos convencionar escrever = q + 1/2, com q = 0, 1, 2, . . .. Teremos aqui n0 = (2q + 1), 1 = = q + 1/2 e 2 = = q 1/2. As solu co es y1 e y2 s ao aquelas dadas em (13.60), (13.61) e (13.62):

y1 (z ) = z q+1/2
n=0

cn (q )z n

y2 (z ) = Ay1 (z ) ln(z ) + z q1/2


n=0

vn (q )z n ,

onde, segundo (13.61), as constantes cn (q ) satisfazem cn (q ) = 1 f n+q+


n1 1 2

m+q+
m=0

1 2

anm + bnm cm (q ) ,

(13.116)

para n 1. Novamente, essas rela co es s ao id enticas ` aquelas de (13.105) com substitu do por q + 1/2 e, assim, suas solu co es s ao c2k (q ) c2k+1 (q ) = =
1 (1)k 1 + q + 2 k ! 22k k + 1 + q + 1 2

c0 (q ) ,

k0.

0,

k0, 1 2q+1/2 1 + q +

onde usamos (1 + q + 1/2) = q !(1/2) e (k + 1 + q + 1/2) = (k + q )!(1/2). Adotando c0 (q ) = chegamos ` a express ao Jq+1/2 (z ) :=
k=0 1 2

z (1)k k ! (k + 1 + q + 1/2) 2

2k+q+1/2

Essa fun ca o representa uma das solu co es da equa ca o de Bessel de ordem q + 1/2 com q = 0, 1, 2, . . . e e denominada fun c ao de Bessel de primeiro tipo e ordem q + 1/2. Passemos agora ` a segunda solu ca o

y2 (z ) = AJq+1/2 (z ) ln(z ) +
n=0

vn (q )z nq1/2 .

Por (13.62), 1 f nq
n1 1 2 m=0

mq

1 2

anm + bnm

vm (q ) ,

1 n 2q ,

vn (q ) =

arbitr ario , 1 f nq 1 2

n = 2q + 1 ,

n1

Agn2q1 +
m=0

mq

1 2

anm + bnm vm (q )

n > 2q + 1,

JCABarata. Curso de F sica-Matem atica

Vers ao de 29 de janeiro de 2013.

Cap tulo 13

603/2069

onde, 1 A = c0 (q ) (2q + 1) Para 1 n 2q tem-se Por em, v1 (q ) = 1 f(1 2 q) 0q 1 2 a1 + b1 v0 (q ) = 0 ,


2q

m=0

mq

1 2

a2q+1m + b2q+1m vm (q )

(13.117)

vn (q ) =

1 vn2 (q ) . f (n q 1 2)

(13.118)

pois a1 = b1 = 0. Conjuntamente com (13.118), isso diz-nos que vn (q ) = 0 para todo n mpar com 1 n 2q . A import ancia dessa observa ca o reside no seguinte. Por (13.117) v e-se facilmente que A = 1 v2q1 (q ) . c0 (q ) (2q + 1)

Portanto, tem-se no caso presente que A = 0 e, assim, a segunda solu ca o e livre de singularidades logar tmicas. Al em disso, com A = 0 as express oes recursivas para vn (q ) simplicam-se para n1 1 1 mq 1 n 2q , anm + bnm vm (q ) , 1 2 f n q 2 m=0 (13.119) vn (q ) = arbitr ario , n = 2q + 1 , n1 1 1 mq anm + bnm vm (q ) , n > 2q + 1. f nq 1 2
2 m=0

Como j a vimos, para 1 n 2q os vn (q ) com n mpar s ao nulos. Como v2q+1 e arbitr ario, e conveniente escolh e-lo igual a zero tamb em. Com isso, as rela co es (13.119) cam id enticas ` aquelas de (13.105) com substitu do por (q + 1/2) e, assim, suas solu co es s ao v2k (q ) v2k+1 (q ) Adotando v0 (q ) = = =
1 (1)k 1 q 2 k ! 22k k + 1 q 1 2

v0 (q ) ,

k0.

0,

k0, 1 2q1/2 1 q
1 2

chagamos ` a seguinte express ao: Jq1/2 (z ) = (1)k k! k + 1 q k=0


1 2

z 2

2kq1/2

Essa fun ca o representa uma segunda solu ca o da equa ca o de Bessel de ordem q + 1/2 com q = 0, 1, 2, . . . e e denominada fun c ao de Bessel de primeiro tipo e ordem (q + 1/2). Conclu mos, assim, que a solu ca o geral da equa ca o de Bessel de ordem q + 1/2 com q = 0, 1, 2, 3, . . ., e 1 Jq+1/2 (z ) + 2 Jq1/2 (z ) , onde 1 e 2 s ao constantes arbitr arias.

JCABarata. Curso de F sica-Matem atica

Vers ao de 29 de janeiro de 2013.

Cap tulo 13

604/2069

Podemos denir tamb em as fun c oes de Neumann de ordem q + 1/2 em analogia com (13.107), mas aqui, tem-se Nq+1/2 (z ) := Jq+1/2 (z ) cos((q + 1/2) ) Jq1/2 (z ) = (1)q+1 Jq1/2 (z ) . sen ((q + 1/2) ) (13.120)

De qualquer forma, a solu ca o geral da equa ca o de Bessel de ordem q + 1/2 com q = 0, 1, 2, 3, . . ., e 1 Jq+1/2 (z ) + 2 Nq+1/2 (z ) , onde 1 e 2 s ao constantes arbitr arias. O estudante e convidado a constatar que Jq+1/2 (z ) e uma fun ca o anal tica para todo z C, z = 0, mas em z = 0 q+1/2 possui uma singularidade como z , que e uma singularidade do tipo ponto ramica ca o (de grau 2). Paralelamente, Jq1/2 (z ) (e, portanto, Nq+1/2 (z )) e anal tica para todo z = 0, mas possui em z = 0 uma singularidade como z q1/2 , que e uma singularidade do tipo ponto ramica ca o (de grau 2). Essas arma co es s ao ilustradas no pr oximo exerc cio. E. 13.10 Exerc cio semi-resolvido. Com q = 0 tem-se pelas nossas deni co es acima

J1/2 (z ) =
k=0

z (1)k k ! (k + 1 + 1/2) 2

2k+1/2

J1/2 (z ) =
k=0

(1)k k! k +

1 2

z 2

2k1/2

Usando as identidades (k + 1 + 1/2) = (3/2) (2k + 1)!! = 2k (2k + 1)!! , 2 2k (2k )!!(2k 1)!! = (2k )! ,
k=0

2k k ! = (2k )!! , (prove-as!) teremos, J1/2 (z ) = z 1/2 e reconhecemos que J1/2 (z ) = Observe ainda que 2
k=0

(2k + 1)!!(2k )!! = (2k + 1)! ,

(1)k z 2k+1 , (2k + 1)!

J1/2 (z ) = z 1/2

(1)k 2k z , (2k )!

2 sen (z ) z 1/2

J1/2 (z ) = 2 sen (z ) , z

2 cos(z ) . z 1/2

(13.121)

J1/2 (z ) = z 1/2 sendo que


sen (z ) z

Complete os detalhes faltantes de todos os c alculos indicados acima.

e uma fun c ao anal tica para todo z C, inclusive em z = 0 (por que?).

E. 13.11 Exerc cio. Verique por c alculo expl cito que as fun co es sen (z )/z 1/2 e cos(z )/z 1/2 s ao, de fato, solu co es da equa c ao de Bessel de ordem = 1/2. Para futura refer encia, reunimos nossos resultados sobre as solu co es da equa ca o de Bessel no seguinte teorema: Teorema 13.3 (Solu co es da equa c ao de Bessel) Seja a equa c ao de Bessel de ordem C z 2 y (z ) + zy (z ) + (z 2 2 )y (z ) = 0, com z C. 1. Caso Z duas solu c oes independentes s ao J (z ) e J (z ), onde

J (z ) :=
k=0

(1)k k ! (k + 1 + )

z 2

2k+

(13.122)

JCABarata. Curso de F sica-Matem atica

Vers ao de 29 de janeiro de 2013.

Cap tulo 13

605/2069

Denindo N (z ) := J (z ) cos( ) J (z ) , sen ( )

as fun c oes J (z ) e N (z ) s ao tamb em duas solu c oes independentes. 2. Caso Z podemos, sem perda de generalidade, adotar 0, pois a equa c ao de Bessel e invariante pela mudan ca . Com essa conven c ao, duas solu c oes independentes s ao J (z ) e N (z ), onde

J (z ) :=
k=0

(1)k k ! (k + 1 + )

z 2

2k+

=
k=0

(1)k k ! (k + )!

z 2

2k+

(13.123)

e N (z ) := sendo que h0 := 0 , hn := 1 + 1 1 1 + + + = 2 3 n 2 + ln z 2 J (z ) 1 ( n 1)! z 2 n=0 n! 2


1 2n

(1)n (hn + hn+ ) 1 2 n=0 n! (n + )! 1 , l

z 2

2n+

l=1

n1.

e e a constante de Euler-Mascheroni: := lim (hn ln(n)) 0, 5772156649 . . ..


n

As fun c oes J (z ), C, s ao denominadas fun co es de Bessel de primeiro tipo e ordem , ou simplesmente fun co es de Bessel de ordem . As fun c oes N (z ), C, s ao denominadas fun co es de Bessel de segundo tipo e ordem , ou fun co es de Neumann de ordem .

Coment ario. O caso em que e semi-inteiro est a inclu do no caso 1, acima: Z. At e o momento denimos as fun co es de Bessel J atrav es das express oes (13.122) e (13.123), mas apenas para s que n ao sejam inteiros negativos. A express ao (13.122) cont em uma fun ca o (x) no denominador e (x) diverge se x for inteiro negativo. Por isso, em princ pio (13.122) n ao est a denida para s inteiros negativos. A experi encia mostrou, por em, que e conveniente denir J para s que sejam inteiros negativos atrav es da seguinte express ao: Jm (z ) := (1)m Jm (z ) , (13.124) para todo m N e todo z C. Note que, como a equa ca o de Bessel e invariante pela troca , Jm denida acima e solu ca o da equa ca o de Bessel de ordem m. A conveni encia dessa conven ca o n ao pode ser apreciada no momento, mas ir a manifestar-se quando discutirmos algumas propriedades das fun co es de Bessel na Se ca o 14.2.7, que inicia-se na p agina 672, tais como as rela co es de recorr encia e a fun ca o geratriz. c ao acima vale E. 13.12 Exerc cio. Mostre que com a conven Jm (z ) = Jm (z ), m N0 , Nota sobre as fun co es de Bessel de ordem inteira negativa

Sugest ao: Jm (z ) e uma soma de mon omios da forma z 2k+m e vale (z )2k+m = (1)m z 2k+m .

13.2.4

Equa c oes Relacionadas ` a de Bessel. A Equa c ao de Bessel Esf erica

Diversas equa co es diferenciais podem ser transformadas na de Bessel e podem ter suas solu co es expressas em termos de fun co es de Bessel e de Neumann. Uma classe bastante geral e composta pelas equa co es da forma z 2 y (z ) + (1 2)zy (z ) + 2 2 z 2 + 2 2 2 y (z ) = 0 , (13.125)

JCABarata. Curso de F sica-Matem atica

Vers ao de 29 de janeiro de 2013.

Cap tulo 13

606/2069

com , , e constantes (sendo = 0). Essa equa ca o e por vezes denominada equa c ao de Bessel generalizada e sua solu ca o mais geral e az J (z ) + bz N (z ) , (13.126) onde a e b s ao constantes arbitr arias. co es acima, ou seja, prove que (13.126) e asolu c ao geral de (13.125). Sugest ao: dena E. 13.13 Exerc cio. Prove as arma a fun c ao v por y (z ) =: z v (z ) e, substituindo em (13.125), mostre que v satisfaz a equa c ao de Bessel de ordem . Dois casos particulares de interesse, dentro da classe denida em (13.125), s ao a equa ca o de Airy (que corresponde a = 1/2, = 2/3, = 3/2 e = 1/3) e a equa ca o de Bessel esf erica (que corresponde a = 1/2, = 1, = 1 e = + 1/2). Trataremos desses casos logo abaixo. O estudante deve observar que, caso 2 n ao seja um inteiro positivo ou zero, a equa ca o (13.125) n ao e singular regular em z0 = 0 (compare ` a (13.49)) e, portanto, a ela n ao se aplica o m etodo de Frobenius. A solu ca o dada em (13.126), de fato, n ao e como aquelas obtidas pelo m etodo de Frobenius, que seriam da forma z (z ) ou da forma z ln(z )(z ), para alguma constante e com anal tica em torno de z0 = 0. Por exemplo, tem-se z J (z ) = z 2
+ k=0

(1)k 2k+ k ! (k + 1 + )

z 2

2k

que n ao e da forma z (z ) com anal tica em torno de z0 = 0, pois a s erie do lado direito n ao e uma s erie de pot encias em z . A equa c ao de Airy e a equa c ao de Bessel

Como dissemos acima, v arias equa co es diferenciais podem ser transformadas em equa co es de Bessel. Um exemplo e o da equa ca o de Airy: y (z ) zy (z ) = 0, cujas solu co es foram apresentadas na Se ca o 13.1.4, p agina 576. A maneira mais simples de ver isso e a seguinte18 . Se y e uma solu ca o da equa ca o de Airy, ent ao a fun ca o v (z ) denida por por 2 3/2 y (z ) =: zv 3 z satisfaz a equa ca o de Bessel de ordem = 1/3, como facilmente se constata. E. 13.14 Exerc cio. Verique isso! mos da que as solu co es y (z ) da equa ca o de Airy podem ser escritas como combina co es lineares das fun co es Conclu 2 3/2 3/2 z e zJ1/3 2 . Com efeito, pelas deni co es (13.29)-(13.30) e (13.122) (para = 1/3) pode-se zJ1/3 3 3z facilmente constatar a validade das rela co es Ai(z ) = z 1/2 J1/3 3 z 1/2 J1/3 3 2 3/2 + J1/3 z 3 2 3/2 J1/3 z 3 2 3/2 z 3 2 3/2 z 3 , (13.127)

Bi(z ) =

(13.128)

que permitem expressar as fun co es de Airy Ai e Bi em termos das fun co es J1/3 . co es (13.127)-(13.128) usando (13.29)-(13.30) e (13.122). E. 13.15 Exerc cio. Prove as rela Na Se ca o 19.5.3, p agina 900, veremos uma aplica ca o dessas considera co es sobre as solu co es da equa ca o de Airy. A equa c ao de Bessel esf erica A equa ca o diferencial z 2 y (z ) + 2zy (z ) + (z 2 ( + 1))y (z ) = 0 ,

para z C, com C, constante, e denominada equa c ao de Bessel esf erica de ordem .

A equa ca o de Bessel esf erica surge, por exemplo, quando da resolu ca o da equa ca o de Helmholtz em tr es dimens oes em coordenadas esf ericas (vide Cap tulo 19, p agina 842) e, portanto, e importante para o estudo da propaga ca o de ondas ou de fen omenos de difus ao em tr es dimens oes.
18 Uma

outra maneira usa propriedades de simetria da equa ca o hipergeom etrica conuente.

JCABarata. Curso de F sica-Matem atica

Vers ao de 29 de janeiro de 2013.

Cap tulo 13

607/2069

Se denirmos v (z ) = z 1/2 y (z ), obtemos para v a equa ca o diferencial z 2 v (z ) + zv (z ) + z2 + 1 2


2

v (z ) = 0 ,

1 . Conseq uentemente as solu co es da equa ca o de Bessel que nada mais e que a equa ca o de Bessel usual de ordem + 2 esf erica s ao da forma J+ 1 (z ) N+ 1 (z ) y (z ) = A 2 + B 2 , z z

onde A e B s ao constantes arbitr arias. Em fun ca o disso, denem-se as chamadas fun c oes de Bessel esf ericas de ordem por j (z ) := J 1 (z ) , 2z + 2 (13.129)

e as chamadas fun c oes de Neumann esf ericas de ordem por n (z ) := N 1 (z ) . 2z + 2 (13.130)

bastante claro que as fun E co es n (z ) s ao singulares em z = 0, enquanto que as fun co es j (z ) n ao divergem em z = 0, sendo at e mesmo fun co es inteiras (anal ticas em toda parte) para inteiro n ao-negativo. Um caso de particular interesse e aquele no qual = l N0 . Nesse caso, podemos escrever a solu ca o geral da equa ca o de Bessel esf erica na forma y (z ) = ajl (z ) + bnl (z ) , com a e b constantes arbitr arias, onde jl (z ) := J 1 (z ) , 2 z l+ 2 N 1 (z ) 2 z l+ 2 = e
(13.120)

(13.131) 1 (z ) . J 2 z ( l+ 2 )

nl (z ) := Note que, por (13.121), tem-se j0 (z ) =

(1)l+1

(13.132)

sen (z ) z

n0 (z ) =

cos(z ) . z

(13.133)

Algumas propriedades das fun co es de Bessel esf ericas ser ao estudadas na Se ca o 14.2.8, p agina 688. As primeiras fun co es de Bessel e de Neumann esf ericas encontram-se listadas em (14.238) e (14.239).

13.2.5

Equa c oes Relacionadas ` a de Bessel. A Equa c ao de Bessel Modicada


z 2 y (z ) + zy (z ) (z 2 + 2 )y (z ) = 0 , (13.134)

Uma outra equa ca o diferencial fortemente relacionada ` a de Bessel e a equa c ao de Bessel modicada de ordem :

com z C, onde C e uma constante. Comparando-se a equa ca o de Bessel (13.104), p agina 595, e f acil perceber que a equa ca o modicada pode ser transformada na de Bessel se zermos formalmente na primeira a substitui ca o z iz . De forma direta, e imediato constatar que se y (z ) e uma solu ca o da equa ca o de Bessel, ent ao y (iz ) e uma solu ca o da equa ca o de Bessel modicada. Conclu mos que no caso de n ao-inteiro a solu ca o geral de (13.134) e dada por uma combina ca o linear de J (iz ) e J (iz ) (ou de J (iz ) e N (iz )) e para = n, inteiro, por uma combina ca o linear de Jn (iz ) e Nn (iz ). Isso sugere e justica as deni co es que seguem.

JCABarata. Curso de F sica-Matem atica

Vers ao de 29 de janeiro de 2013.

Cap tulo 13

608/2069

Denem-se as fun c oes de Bessel modicadas de primeira esp ecie e de ordem , denotadas por I (z ), por I (z ) := i J (iz ) = ei/2 J (iz ) , sendo que para = n, inteiro, tem-se In (x) = In (x) = in Jn (iz ) . As fun c oes de Bessel modicadas de segunda esp ecie e de ordem , denotadas por K (z ), s ao denidas por K (z ) := i +1 J (iz ) + iN (iz ) . 2

As fun co es K s ao denominadas por alguns autores fun c oes de Macdonald19 . Advert encia. O estudante deve ser advertido do fato de n ao haver, infelizmente, uniformidade na literatura quanto ` a deni ca o das fun co es K apresentadas acima, pois alguns textos adotam para K uma combina ca o linear das fun co es J (iz ) e N (iz ) com constantes ligeiramente diferentes daquelas de acima. A refer encia [257], por exemplo, multiplica a express ao por cos( ) de modo a fazer com que as fun co es K satisfa cam as mesmas rela co es de recorr encia que as fun co es I . Desastradamente, por em, isso faz com que a express ao se anule se = 1/2 + k , com k Z. A deni ca o que adotamos e a mais comum atualmente e, curiosamente, coincide com a original de Basset20 de 1886. Vide [257] para outros coment arios sobre esse ponto. Note-se que I (z ) e K (z ) s ao linearmente independentes, de modo que a solu ca o geral da equa ca o de Bessel modicada de ordem e uma combina ca o linear aI (z ) + bK (z ), onde a e b s ao constantes. co es acima, E. 13.16 Exerc cio. Mostre que, com as deni K (z ) = I (z ) I (z ) 2 sen ( ) e que K (z ) = K (z ) .

Da representa ca o em s erie (13.122) das fun co es de Bessel, e da deni ca o de I (z ) obtem-se

I (z ) :=
k=0

1 k ! (k + 1 + )

z 2

2k+

(13.135)

de se notar que se > 0 e se e z > 0 ent E ao todos os termos da s erie acima s ao positivos e, portanto, I (z ) > 0. Assim, ao contr ario das fun co s de Bessel, as fun co es de Bessel modicadas I n ao se anulam no eixo real positivo. O mesmo pode ser facilmente provado sobre as fun co es K , as quais divergem em z = 0. Para o caso em que = m N0 , temos

Im (z ) :=
k=0

1 k ! (m + k )!

z 2

2k+m

(13.136)

13.2.6

A Equa c ao de Laguerre
zy (z ) + (1 z )y (z ) + y (z ) = 0 ,

A equa c ao de Laguerre21 e a equa ca o diferencial

com z C, onde C e uma constante.

A equa ca o de Laguerre, e uma parente pr oxima, a equa ca o de Laguerre associada, apresentada na Se ca o 13.3.2, p agina 618, emergem em um dos problemas mais importantes da F sica, a equa ca o de Schr odinger para o atomo de hidrog enio
19 Hector 20 Alfred

Munro Macdonald (18651935). Barnard Basset (18541930). 21 Edmond Nicolas Laguerre (18341886).

JCABarata. Curso de F sica-Matem atica

Vers ao de 29 de janeiro de 2013.

Cap tulo 13

609/2069

em coordenadas esf ericas. Vide Se ca o 19.8, p agina 909. A equa ca o de Laguerre e tamb em um caso particular da equa ca o hipergeom etrica conuente, a ser discutida na Se ca o 13.2.8, p agina 614. Comparando com a forma (13.52), vemos que z0 = 0 e um ponto singular regular da equa ca o, vemos que a(z ) = 1 z e que b(z ) = z . Assim, no presente caso tem-se 1, para n = 0 , para n = 1 , b = . an = n 1, para n = 1 0, para n = 0 ou n 2 0, para n 2

elementar constatar-se que, para essa equa E ca o, = + = 0 e, portanto, estamos no caso 2 do Teorema 13.2 da p agina 583 com f (x) = x2 , 0 = 0, y1 (z ) =
n=0

cn z n

y2 (z ) = y1 (z ) ln(z ) +
n=0

vn z n ,

(13.137)

onde cn = e vn = 1 n2

n1 m=0

manm + bnm cm =

n+1 cn1 , n2

n2,

1 n2 1 n2

n1

2 n 1 cn +

anm cm +
m=0 m=0

manm + bnm vm

2n cn cn1

n+1 vn1 , n2

n 1 ,

(13.138)

Adotando-se c0 = 1, obtem-se para n 1 cn = e y1 (z ) ca y1 (z ) = 1 + (1)n (n!)2 n=1


n1

(1)n (n!)2

n1

l=0

( l) =

(1)n ( + 1) (n!)2 ( n + 1) (1)n ( + 1) zn . 2 ( n + 1) ( n !) n=1

l=0

( l)

zn = 1 +

(13.139)

A situa ca o de maior interesse em F sica e aquela na qual e um inteiro n ao-negativo: = m N0 . A raz ao disso ser a explicada detalhadamente no Ap endice 13.E, p agina 626, mas adiantamos que nos casos em que n ao e um inteiro positivo a solu ca o y1 cresce muito rapidamente (exponencialmente) quando z e restrito ao semi-eixo real positivo. Esse comportamento e inadequado em v arias aplica co es, por exemplo no cl assico problema do atomo de hidrog enio da Mec anica Qu antica, o que leva ao descarte de tais solu co es. J a no caso em que e um inteiro n ao-negativo, = m N0 , a solu ca o dada em (13.139) reduz-se a um polin omio de grau m: y1 (z ) = 1 + (1)n (n!)2 n=1
m n1 l=0

(m l)

zn

= 1+
m

(1)n m! zn 2 (m n)! ( n !) n=1 m n zn .

(1)n n! n=0

JCABarata. Curso de F sica-Matem atica

Vers ao de 29 de janeiro de 2013.

Cap tulo 13

610/2069

Os chamados polin omios de Laguerre, denotados por Lm (z ), s ao denidos como m! vezes o polin omio acima22 :
m

Lm (z ) := Os quatro primeiros s ao L0 (z ) = 1, L1 (z ) = 1 z,

n=0

(1)n

m! n!

m n

zn .

(13.140)

L2 (z ) = 2 4z + z 2 ,

L3 (z ) = 6 18z + 9z 2 z 3 .

f E acil provar, tamb em, que a seguinte express ao e v alida (vide p agina 666): Lm (z ) = ez dm z m e z . dz m (13.141)

Os polin omios de Laguerre Lm (z ) s ao, portanto, uma das solu co es da equa ca o de Laguerre (com = m) zy (z ) + (1 z )y (z ) + my (z ) = 0 , com z C, onde m N0 . De acordo com (13.137), uma segunda solu ca o e dada na forma

(13.142)

y2 (z ) = Lm (z ) ln(z ) +
n=0

vn z n ,

onde os coecientes vn s ao dados em (13.138) em termos dos coecientes cn dos polin omios de Laguerre. Ap os c alculos um tanto ma cantes, chega-se ` a seguinte express ao:
m

y2 (z ) = Lm (z ) ln(z ) +
k=1

m! (1) k!
k

m k

(hmk hm 2hk ) z + (1)

m k=1

(k 1)! z m +k , (m + 1)2 (m + 2)2 (m + k )2

onde hn est a denido em (13.109)-(13.110). E. 13.17 Exerc cio. Mostre isso. Sugest ao: tire uma tarde livre. ao deseje fazer o exerc cio anterior, poder a contentar-se com a tarefa mais simples de E. 13.18 Exerc cio. Caso o leitor n vericar que a express ao acima e, de fato, uma solu c ao de (13.142). Essa segunda solu ca o e raramente empregada em problemas de F sica, especialmente devido ` a singularidade logar tmica que apresenta. Mais propriedades dos polin omios de Laguerre ser ao estudadas na Se ca o 14.2.5, p agina 665.

13.2.7

A Equa c ao Hipergeom etrica


z (1 z )y (z ) + [ (1 + + )z ]y (z ) y (z ) = 0 , (13.143)

A equa ca o diferencial para z C e com , e C constantes, e denominada equa c ao hipergeom etrica, ou equa c ao de Gau23 , quem a primeiro estudou. A raz ao do interesse nessa equa ca o reside em tr es fatos. Primeiro, a equa ca o hipergeom etrica e (a menos de multiplica ca o trivial por uma constante) a u nica equa ca o linear homog enea de segunda ordem com apenas tr es pontos singulares regulares em 0, 1 e (vide discuss ao ` a p agina 541). Sabe-se, ademais, (vide discuss ao da Se ca o 12.8.3, p agina
22 O fator de normaliza ca o m! tem origem hist orica. O leitor deve ser advertido do fato, j a lamentado p aginas acima, que em alguns textos outra normaliza ca o e empregada. 23 Johann Carl Friedrich Gau (17771855). Um dos maiores e mais inuentes matem aticos de todos os tempos, Gau dedicou-se tamb em intensamente a problemas de F sica, Astronomia, Matem atica Aplicada e mesmo Engenharia ( e um dos co-inventores do tel egrafo) e encontrou as equa co es hipergeom etricas em estudos de Geodesia, assunto a que se dedicou quando da constru ca o das primeiras linhas f erreas da Alemanha. Seus trabalhos nessa area tamb em inspiraram uma das suas muitas contribui co es importantes ` a matem atica pura: a formula ca o de geometrias n ao-Euclidianas.

JCABarata. Curso de F sica-Matem atica

Vers ao de 29 de janeiro de 2013.

Cap tulo 13

611/2069

554) que toda equa ca o Fuchsiana com tr es pontos singulares pode ser transformada em uma equa ca o hipergeom etrica. Segundo, h a v arias equa co es diferenciais de interesse que podem ser transformadas em equa co es hipergeom etricas e, com isso, pode-se estudar certas propriedades de v arias fun co es especiais, tais como seu comportamento assint otico, a partir das propriedades correspondentes de fun co es hipergeom etricas. Terceiro, suas solu co es possuem muitas simetrias. A equa ca o hipergeom etrica e uma das equa co es diferenciais ordin arias mais estudadas, sendo suas solu co es riqu ssimas em propriedades. Sua abordagem completa est a muito al em das pretens oes destas Notas e, para um tratamento detalhado, recomendamos as refer encias [110], [224], [257], [151], [107] e outras. Propriedades combinat orias envolvendo as s eries hipergeom etricas e suas generaliza co es podem ser encontradas em [84]. Vamos aqui apresentar as solu co es da equa ca o hipergeom etrica (13.143) em termos de expans oes em torno de seu ponto singular regular z0 = 0. O leitor poder a encontrar em [224] solu co es de (13.143) expressas como expans oes em torno dos outros pontos singulares regulares z0 = 1 e z0 = . O interesse nessas u ltimas expans oes e um tanto menor, especialmente pois as mesmas podem ser expressas em termos das solu co es obtidas em torno de z0 = 0. Reescrevemos (13.143) na forma b(z ) a(z ) (13.144) y (z ) + 2 y (z ) = 0 , y (z ) + z z sendo a(z ) e b(z ) anal ticas em |z | < 1, a saber, a(z ) = (1 + + )z = 1z z = 1z

an z n = +
n=0 n=1

( 1 )z n ,

b(z ) = A equa ca o indicial, neste caso, e

bn z n =
n=0

n=1

( )z n .

f (x) = x(x 1) + x = x(x + 1) = 0 e temos, portanto, os ndices dados por = 1 e + = 0 .

H a, assim, tr es casos a considerar: 1. 1 Z, ou seja, Z. 2. = 1. 3. 1 Z \ {0}, ou seja, Z mas = 1. Caso 1. 1 Z, ou seja, Z. Aqui, de acordo com (13.54) e (13.55), as solu co es s ao

y1 (z ) = z 1
n=0

cn z n

y2 (z ) =
n=0

dn z n ,

(13.145)

onde cn = 1 f (1 + n)
n1 m=0

(m + 1 )anm + bnm cm ,

dn =

1 f (n)

n1

manm + bnm dm ,
m=0

para todo n 1. Nesse caso, por em, n ao e t ao simples resolver recursivamente essas equa co es, pelo menos na maneira muito mais f como est ao expressas acima. E acil obter as rela co es recursivas de outra forma: inserindo (13.145) na equa ca o diferencial ainda na forma (13.143). Com esse procedimento, come cando pela solu ca o y2 (z ), obtem-se alegremente para os coecientes dn a seguinte rela ca o recursiva: dn+1 = para todo n 0. E. 13.19 Exerc cio importante. Verique! ( + n)( + n) dn , (n + 1)( + n) (13.146)

JCABarata. Curso de F sica-Matem atica

Vers ao de 29 de janeiro de 2013.

Cap tulo 13

612/2069

Convencionando-se tomar d0 = 1, chegamos a dn = onde, para x C e n N0 , x(x + 1) (x + n 1) = 1,


n1 l=0

()n ( )n , n!( )n

n1,

(x + l) , n 1 , (13.147) n = 0,

(x)n :=

s ao os denominados s mbolos de Pochhammer24 . Quando x n ao e um inteiro negativo ou zero, podemos escrever (x)n = Com isso, obtemos para a solu ca o y2 a express ao F (, , , z ) := 1 + ()n ( )n n ( + n)( + n) z n ( ) z = . n!( )n ()( ) n=0 ( + n) n! n=1

(x + n) . (x)

(13.148)

Essa fun ca o, introduzida por Gau em cerca de 1812, e denominada fun c ao hipergeom etrica, denomina ca o aparentemente criada por Kummer25 em 1836. Contribu ram ` a teoria das fun co es hipergeom etricas nomes como Euler, Gau, Kummer e Riemann. Na literatura F (, , , z ) e muitas vezes denotada por 2 F1 (, , , z )26 . Repetindo considera co es anteriores, F (, , , z ) e anal tica como fun ca o de z pelo menos na regi ao |z | < 1. No caso em que ou s ao inteiros n ao-positivos, e f acil ver que F (, , , z ) reduz-se a um polin omio e e, portanto, anal tica em toda parte. Exceto nesses casos, a s erie que dene F (, , , z ) e divergente para |z | > 1, como se v e pelo teste da raz ao, pois ()n+1 ( )n+1 n+1 | + n | | + n | (n+1)!( )n+1 z |z | , = ()n ( )n n (n + 1) | + n| z
n!( )n

que, para n grande, aproxima-se de |z | > 1. Casualmente, o mesmo argumento prova converg encia absoluta da s erie hipergeom etrica (13.148) para |z | < 1. Fazemos ainda notar que a express ao acima para F (, , , z ) est a denida mesmo para o caso em que e um inteiro positivo e, portanto, representa uma solu ca o da equa ca o hipergeom etrica naquele caso. Para nulo ou um inteiro negativo, digamos = m, o denominador ( )n anula-se para n > m e a express ao para F (, , , z ) deixa de fazer sentido.

Para obtermos a outra solu ca o inserimos y1 de (13.145) na equa ca o diferencial ainda na forma (13.143) e obtemos alegremente para os coecientes cn a rela ca o cn+1 = para todo n 0. E. 13.20 Exerc cio importante. Verique!
August Pochhammer (18411920). Eduard Kummer (18101893). 26 A explica ca o da nota ca o 2 F1 e a seguinte: o 2 ` a esquerda indica a presen ca de dois s mbolos de Pochhammer no numerador dos termos da s erie hipergeom etrica (13.148). O 1 ` a direita indica a presen ca de um s mbolo de Pochhammer no denominador. H a generaliza co es da s erie (13.148) que denem as chamadas fun c oes hipergeom etricas generalizadas, denotadas por k Fl , e que cont em k s mbolos de Pochhammer no numerador e l no denominador (vide e.g. [84]). Mais abaixo encontraremos as fun co es hipergeom etricas conuentes, que s ao do tipo 1 F1 .
25 Ernst 24 Leo

(n + + 1 )(n + + 1 ) cn , (n + 1)(n + 2 )

JCABarata. Curso de F sica-Matem atica

Vers ao de 29 de janeiro de 2013.

Cap tulo 13

613/2069

Alguns segundos de contempla ca o nos levam a concluir que essas rela co es s ao id enticas ` aquelas de (13.146), desde que l a fa camos as seguintes modica co es: + 1 , + 1 e 2 . Por tr as dessa aparente coincid encia residem propriedades de simetria da equa ca o hipergeom etrica. O leitor poder a encontrar essa discuss ao nos textos supra-citados. Assim, tomando-se tamb em c0 = 1, conclu mos que a outra solu ca o e z 1 F ( + 1 , + 1 , 2 , z ) . Fazemos ainda notar que F ( + 1 , + 1 , 2 , z ) est a denida mesmo para o caso em que e um inteiro n ao-positivo e, portanto, z 1 F ( + 1 , + 1 , 2 , z ) representa uma solu ca o da equa ca o hipergeom etrica naquele caso. Resumindo nossas conclus oes, para o caso Z a solu ca o geral da equa ca o hipergeom etrica (13.143) expressa em termos de uma expans ao em torno do ponto singular regular z0 = 0 e A1 z 1 F ( + 1 , + 1 , 2 , z ) + A2 F (, , , z ) . onde A1 e A2 s ao constantes arbitr arias. Caso 2. = 1. Aqui = + = 0 = 0. Nesse caso a primeira solu ca o e da forma y1 (z ) = cn+1 = para todo n 0. Assim, a primeira solu ca o e
n=0 cn

z n e, de modo an alogo, obtemos (13.149)

( + n)( + n) cn , (n + 1)2

1 zn ()n ( )n n z = . ( + n)( + n) F (, , 1, z ) = 1 + 2 (n!) ()( ) n=0 (n!)2 n=1 Pelo mesmo argumento de acima, a expans ao em s erie do lado direito converge para |z | < 1 e diverge para |z | > 1. Pelo Teorema 13.2, p agina 583, a segunda solu ca o tem a forma F (, , 1, z ) ln(z ) +
n=0

vn z n ,

com os vn dados em (13.58) em termos dos cn de acima. A express ao que se obtem e um tanto complexa e evitamos coloc a-la aqui. O leitor poder a encontr a-la, por exemplo, em [224]. Caso 3. 1 Z \ {0}, ou seja, Z mas = 1. No caso a, = m, com m > 1 inteiro. Aqui tem-se n0 = m 1, 1 = + = 0 e 2 = = 1 m. Como j a observamos acima, uma solu ca o e dada por F (, , m, z ). Uma segunda solu ca o ser a da forma

H a dois casos a distinguir: a. > 1 e b. 0.

AF (, , m, z ) ln(z ) + z 1m
n=0

vn z n ,

com os vn e A dados como em (13.62) e (13.63) a partir dos coecientes cn de F (, , m, z ). Novamente, a express ao que se obtem e complexa e remetemos o estudante a, e.g., [224]. No caso b, = m, com m 0 inteiro. Aqui tem-se n0 = m + 1, 1 = = 1 + m e 2 = + = 0. Como j a observamos acima, uma solu ca o e dada por z 1+m F ( + 1 + m, + 1 + m, 2 + m, z ). Uma segunda solu ca o ser a da forma

Az 1+m F ( + 1 + m, + 1 + m, 2 + m, z ) ln(z ) +
n=0 1+m

vn z n ,

com os vn e A dados como em (13.62) e (13.63) a partir dos coecientes cn de z F ( + 1 + m, + 1 + m, 2 + m, z ). Novamente, a express ao que se obtem e complexa e remetemos o estudante a, e.g., [224]. Com isso encerramos nossa breve excurs ao ` as fun co es hipergeom etricas e remetemos o estudante interessado em um maior aprofundamento ` a literatura supra-citada.

JCABarata. Curso de F sica-Matem atica

Vers ao de 29 de janeiro de 2013.

Cap tulo 13

614/2069

13.2.8

A Equa c ao Hipergeom etrica Conuente


zy (z ) + [ z ]y (z ) y (z ) = 0 , (13.150)

A equa ca o diferencial para z C e com e C constantes, e denominada equa c ao hipergeom etrica conuente ou equa c ao de Kummer. A mesma pode ser obtida da equa ca o hipergeom etrica por um procedimento de limite no qual a singularidade regular de z0 = 1 daquela equa ca o e feita imergir (conuir, da o nome) na singularidade regular de z0 = . Esse processo pode ser descrito da seguinte forma. Fa camos na equa ca o hipergeom etrica z (1 z )y (z ) + [ (1 + + )z ]y (z ) y (z ) = 0 a mudan ca de vari aveis = z . A mesma assume a forma (verique!) 1 d2 y + d 2 ++1 dy y = 0 . d

Tomando-se agora o limite | | obtemos a forma (13.150). Vide, e.g., [224] ou [110]. A equa ca o hipergeom etrica conuente possui uma singularidade regular em z0 = 0 e uma irregular em z0 = (vide discuss ao ` a p agina 542). Assim como no caso da equa ca o hipergeom etrica, h a v arias equa co es diferenciais de interesse que podem ser transformadas em equa co es hipergeom etricas conuentes. Os exemplos mais evidentes s ao a equa ca o de Laguerre, Se ca o 13.2.6, p agina 608, que corresponde a = 1 e = , e a equa ca o de Laguerre associada, Se ca o 13.3.2, p agina 618, que corresponde a = m + 1 e = (n m). Um outro exemplo e a equa ca o de Hermite, equa ca o (13.15), p agina 574, que pode ser transfomada em uma equa ca o hipergeom etrica conuente denindo-se w = z 2 e v (w) = y (z ). Com isso, (13.15) transforma-se em 1 w v (w) + v (w) = 0 , (13.151) wv (w) + 2 4 (verique!) que e uma equa ca o hipergeom etrica conuente com =
1 2

Pode-se, portanto, estudar propriedades de v arias fun co es especiais, tais como sua estrutura de singularidades ou seu comportamento assint otico, a partir das propriedades correspondentes de fun co es hipergeom etricas conuentes. Para a equa ca o hipergeom etrica conuente tem-se y (z ) + [ z ] z y (z ) 2 y (z ) = 0 z z

e = 4.

e assim, comparando com a forma padr ao (13.49), temos a(z ) = z, Logo, , para n = 0 para n = 1 para n 2 , bn = e b(z ) = z . ,

an =

para n = 1 para n = 0 ou n 2

A equa ca o indicial e, portanto, cujas ra zes s ao

1, 0,

0,

f (x) = x(x + 1) , = 1 e + = 0 ,

Caso 1. 1 Z, ou seja, Z.

tal como para a equa ca o hipergeom etrica. H a, assim, tr es casos a considerar: 1. 1 Z, ou seja, Z. 2. = 1. 3. 1 Z \ {0}, ou seja, Z mas = 1.

JCABarata. Curso de F sica-Matem atica

Vers ao de 29 de janeiro de 2013.

Cap tulo 13

615/2069

Aqui, de acordo com (13.54) e (13.55), as solu co es s ao


y1 (z ) = z onde cn = 1 f (1 + n)
n1 m=0

1 n=0

cn z

y2 (z ) =
n=0

dn z n ,

(13.152)

(m + 1 )anm + bnm cm , n+ cn1 , n(n + 1 ) cn = ( + 1 )n c0 , n!(2 )n

dn =

1 f (n)

n1

manm + bnm dm ,
m=0

para todo n 1. Assim, o que conduz a

cn =

dn =

n+1 dn1 , n(n + 1) ()n d0 , n!( )n

dn =

(13.153)

Tomando d0 = 1 a solu ca o y2 assume a forma


1 F1 (,

, z ) := 1 +

( + n) z n ( ) ()n n z = . n!( )n () n=0 ( + n) n! n=1

(13.154)

Esta fun ca o e denominada fun c ao hipergeom etrica conuente ou, por vezes, fun c ao de Kummer. E. 13.21 Exerc cio. Prove, usando diretamente as deni co es, a seguinte rela c ao entre as fun co es hipergeom etricas conuentes e as fun co es hipergeom etricas:
1 F1 (,

, z ) =

| |

lim F

, , ,

Aplicando o teste da raz ao ` a s erie de (13.154) temos


()n+1 n+1 (n+1)!( )n+1 z ()n n n!( )n z

| + n | |z | (n + 1) | + n|

e vemos que a mesma converge absolutamente para todo z C, pois para cada z xo o lado direito torna-se menor que 1 para n grande o suciente. Assim, 1 F1 (, , z ) e anal tica para todo z C.

Fazemos ainda notar que a express ao acima para 1 F1 (, , z ) est a denida mesmo para o caso em que e um inteiro positivo e, portanto, representa uma solu ca o da equa ca o hipergeom etrica conuente naquele caso. Para nulo ou um inteiro negativo, digamos = m, o denominador ( )n anula-se para n > m e a express ao para F (, , z ) deixa de fazer sentido.

Passemos agora ` a solu ca o y1 . Alguns segundos de contempla ca o das express oes de (13.153) conduzem-nos ` a percep ca o que a rela ca o entre cn e c0 equivale ` a rela ca o entre dn e d0 com a troca + 1 e 2 (tal como se fez no caso da equa ca o hipergeom etrica, acima). Assim, convencionando-se tamb em c0 = 1, tem-se que a solu ca o y1 (z ) e dada por z 1 1 F1 ( + 1 , 2 , z ) . Fazemos ainda notar que 1 F1 ( + 1 , 2 , z ) est a denida mesmo para o caso em que e um inteiro n ao-positivo e, portanto, z 1 1 F1 ( + 1 , 2 , z ) representa uma solu ca o da equa ca o hipergeom etrica conuente naquele caso. Resumindo, para o caso Z a solu ca o geral da equa ca o hipergeom etrica conuente (13.150) e A1 z 1 1 F1 ( + 1 , 2 , z ) + A2 1 F1 (, , z ) , onde A1 e A2 s ao constantes arbitr arias.

JCABarata. Curso de F sica-Matem atica

Vers ao de 29 de janeiro de 2013.

Cap tulo 13

616/2069

Caso 2. = 1. Esse e o caso da equa ca o de Laguerre. Aqui = + = 0 = 0. Nesse caso a primeira solu ca o e da forma y1 (z ) = cn+1 = para todo n 0. Assim, a primeira solu ca o e
1 F1 (, 1, z ) = 1 + n=0 cn

z n e, de modo an alogo, obtemos (13.155)

( + n) cn , (n + 1)2

()n n 1 zn z = . ( + n) 2 (n!) () n=0 (n!)2 n=1

Pelo Teorema 13.2, p agina 583, a segunda solu ca o tem a forma


1 F1 (,

1, z ) ln(z ) +
n=0

vn z n ,

com os vn dados em (13.58) em termos dos cn de acima. A express ao que se obtem e um tanto complexa e evitamos coloc a-la aqui. Caso 3. 1 Z \ {0}, ou seja, Z mas = 1. Esse e o caso da equa ca o de Laguerre associada. H a dois casos a distinguir: a. > 1 e b. 0.

No caso a, = m, com m > 1 inteiro. Aqui tem-se n0 = m 1, 1 = + = 0 e 2 = = 1 m. Como j a observamos acima, uma solu ca o e dada por 1 F1 (, m, z ). Uma segunda solu ca o ser a da forma

A 1 F1 (, m, z ) ln(z ) + z 1m
n=0

vn z n ,

com os vn e A dados como em (13.62) e (13.63) a partir dos coecientes cn de 1 F1 (, m, z ). Novamente, a express ao que se obtem e complexa e a omitimos aqui. No caso b, = m, com m 0 inteiro. Aqui tem-se n0 = m + 1, 1 = = 1 + m e 2 = + = 0. Como j a observamos acima, uma solu ca o e dada por z 1+m 1 F1 ( + 1 + m, 2 + m, z ). Uma segunda solu ca o ser a da forma

Az 1+m 1 F1 ( + 1 + m, 2 + m, z ) ln(z ) +
n=0

vn z n ,

com os vn e A dados como em (13.62) e (13.63) a partir dos coecientes cn de z 1+m 1 F1 ( + 1 + m, 2 + m, z ). Novamente, a express ao que se obtem e complexa e e omitida aqui. Com isso encerramos nossa breve excurs ao ` as fun co es hipergeom etricas conuentes. Para um tratamento extensivo da equa ca o hipergeom etrica conuente e propriedades de suas solu co es, vide [221], [110] ou [257].

13.3

Algumas Equa co es Associadas

Algumas das equa co es tratadas acima possuem parentes pr oximos com os quais se relacionam amistosamente. Vamos estudar algumas delas.

13.3.1

A Equa c ao de Legendre Associada


2 y (z ) = 0 . 1 z2

A equa c ao de Legendre associada e equa ca o diferencial (1 z 2 )y (z ) 2zy (z ) + ( + 1)y (z ) (13.156)

JCABarata. Curso de F sica-Matem atica

Vers ao de 29 de janeiro de 2013.

Cap tulo 13

617/2069

Como e f acil de se constatar, os pontos 1 s ao pontos singulares regulares da equa ca o de Legendre associada. Repare tamb em que para = 0 recupera-se a equa ca o de Legendre usual (1 z 2 )y (z ) 2zy (z ) + ( + 1)y (z ) = 0 . (13.157)

O principal interesse na equa ca o (13.156) se d a no caso em que e um n umero inteiro, = m Z, situa ca o que corresponde ` a maioria das aplica co es. Nesse caso, um truque feliz permite-nos encontrar as solu co es sem termos de recorrer ao m etodo de Frobenius. Tudo come ca com a observa ca o que a equa ca o de Legendre usual e a equa ca o de Legendre associada podem ser transformadas em uma mesma equa ca o. Se em (13.156) (j a adotando = m Z) zermos a substitui ca o y (z ) = (1 z 2 )m/2 v (z ), obtemos para v a equa ca o (1 z 2 )v (z ) 2(m + 1)z v (z ) + ( + 1) m(m + 1) v (z ) = 0 . E. 13.22 Exerc cio importante. Mostre isso. Sugest ao: um pouco de paci encia. Se, por outro lado, tomarmos a equa ca o (13.157) e a derivarmos m vezes, obtemos (1 z 2 ) y (m)

(13.158)

(z ) 2(m + 1)z y (m)

(z ) + ( + 1) m(m + 1)

y (m) (z ) = 0 .

(13.159)

E. 13.23 Exerc cio importante. z 2 )y (z ) e


d dz m
m

Mostre isso. Sugest ao: use a regra de Leibniz para calcular as derivadas

dm dz m

(1

zy (z ) .

Comparando (13.158) com (13.159), constatamos que ambas s ao a mesma equa ca o. Com isso, vemos que se yL ea solu ca o geral da equa ca o de Legendre e yLa e a solu ca o geral da equa ca o de Legendre associada, ent ao (1 z 2 )m/2 yLa (z ) (m ) e yL (z ) devem ser proporcionais, j a que obedecem ` a mesma equa ca o (13.158). Com isso, obtemos que a solu ca o geral da equa ca o de Legendre associada pode ser obtida da solu ca o geral da equa ca o de Legendre por yLa (z ) = km (1 z 2 )m/2 yL (z ) , km sendo constantes de normaliza ca o a serem convencionadas. Coloquemo-nos agora a quest ao: qual solu ca o yL da equa ca o de Legendre devemos adotar? Isso certamente depende do tipo de problema considerado, mas na maioria das aplica co es procuramos resolver a equa ca o de Legendre associada no intervalo [1, 1] e procuramos solu co es que sejam nitas em todo esse intervalo, incluindo as bordas 1. Ora, j a vimos que as u nicas solu co es da equa ca o de Legendre usual que permanecem limitadas nos extremos 1 (assim como suas derivadas) s ao os polin omios de Legendre Pl (z ), os quais ocorrem como solu ca o apenas no caso = l, um inteiro n ao-negativo. Obtemos, assim, que as solu co es de interesse da a ca o de Legendre associada que s ao limitadas em todo o intervalo fechado [1, 1] ocorrem para = l, um inteiro n ao-negativo, e s ao dadas por Plm (z ) := (1 z 2 )m/2 dm Pl (z ) , dz m (13.160)
(m )

claro que P m (z ) onde Pl e o polin omio de Legendre de grau l. E e nulo se m > l (pois Pl e um polin omio de grau l). l As fun co es Plm denidas acima s ao denominadas polin omios de Legendre associados, ainda que n ao sejam realmente polin omios em z no caso em que m e mpar (devido ao fator (1 z 2 )m/2 )27 e desempenham um papel importante na resolu ca o de equa co es diferenciais parciais em 3 dimens oes em coordenadas esf ericas, tais como a equa ca o de Laplace e de Helmholtz. A eles est ao intimamente relacionados as chamadas fun co es harm onicas esf ericas, das quais falaremos na Se ca o 14.2.2, p agina 646, e que desempenham um papel na Mec anica Qu antica (orbitais at omicos), na Teoria de Grupos (representa co es do grupo SO(3)), no Eletromagnetismo (emiss ao de ondas eletromagn eticas por antenas) etc.
omio no entanto, substituirmos z por cos , com 0 , o que costumeiramente se faz em aplica co es, Plm (cos ) torna-se um polin 2 m/ 2 m trigonom etrico, ou seja, um polin omio em cos e sen , j a que (1 z ) torna-se ( sen ( )) . Essa e a raz ao dessa nomenclatura. Vide express ao (14.63), p agina 649.
27 Se,

JCABarata. Curso de F sica-Matem atica

Vers ao de 29 de janeiro de 2013.

Cap tulo 13

618/2069

As fun co es Plm est ao denidas acima para l inteiro n ao-negativo, ou seja l = 0, 1, 2, 3, . . ., e m inteiro com 0 m l (pois para m > l o lado direito de (13.160) anula-se). Cada Plm e solu ca o da equa ca o de Legendre associada (1 z 2 )y (z ) 2zy (z ) + l(l + 1)y (z ) m2 y (z ) = 0 . 1 z2 (13.161)

Na Se ca o 14.2.1, que se inicia ` a p agina 642, mostraremos que os polin omios de Legendre podem ser escritos como Pl (z ) = 2l 1 dl (z 2 1)l , l! dz l

express ao essa conhecida como f ormula de Rodrigues para os polin omios de Legendre. Assim, obtemos Plm (z ) = 2l dl+m 1 (1 z 2 )m/2 l+m (z 2 1)l , l! dz (13.162)

express ao v alida para 0 m l, com l um inteiro n ao-negativo: l = 0, 1, 2, 3, . . .. Caso m > l, o lado direito se anula. Um ponto interessante, por em, e que a express ao do lado direito de (13.162) est a bem denida para quaisquer l e m com l + m 0, ou seja, tamb em para ms negativos tais que m l. Assim, (13.162) est a denida para todo m inteiro com l m l28 . Da express ao (13.162), entendida para todo l inteiro n ao-negativo e l m l, e poss vel mostrar que Plm (z ) = (1)m (l m)! m P (z ) . (l + m)! l

Essa rela ca o, que e relevante para as chamadas fun co es harm onicas esf ericas, mostra que Plm (z ) e tamb em solu ca o da m equa ca o de Legendre associada (13.161), por ser proporcional a Pl (z ). Trataremos disso na Se ca o 14.2.2, p agina 646, onde outras propriedades dos polin omios de Legendre associados ser ao apresentadas e sua rela ca o com as harm onicas esf ericas ser a discutida. Os primeiros polin omios de Legendre associados s ao
0 P0 (z ) = 1 ; 2 P2 (z ) =

1 1 P1 (z ) = (1 z 2 )1/2 , 2 1 z (1 z 2)1/2 , 2
0 P2 (z ) =

0 P1 (z ) = z ,

1 P1 (z ) = (1 z 2 )1/2 ; 2 P2 (z ) = 3(1 z 2) .

1 (1 z 2) , 8

1 P2 (z ) =

1 (3z 2 1) , 2

1 P2 (z ) = 3z (1 z 2)1/2 ,

E. 13.24 Exerc cio. Verique!

13.3.2

A Equa c ao de Laguerre Associada


xy + (m + 1 x)y + (n m)y = 0 . (13.163)

A equa c ao de Laguerre associada e a equa ca o diferencial

O principal interesse nessa equa ca o reside no caso onde m e n s ao inteiros satisfazendo 0 m n. Como o leitor facilmente constata, trata-se de um caso particular da equa ca o hipergeom etrica conuente (13.150). A equa ca o de Laguerre associada surge da equa ca o de Schr odinger para o atomo de hidrog enio quando a mesma e resolvida pelo m etodo de separa ca o de vari aveis em coordenadas esf ericas. A solu ca o dessa equa ca o pode ser obtida diretamente da solu ca o da equa ca o de Laguerre usual xy + (1 x)y + ny = 0 pois esta, quando diferenciada m vezes em rela ca o ` a x, transforma-se exatamente na equa ca o (13.163).
28 De passagem, comentamos que a rela ca o l m l desempenha um papel na teoria do momento angular na Mec anica Qu antica, mas isso n ao e nosso assunto aqui.

(13.164)

JCABarata. Curso de F sica-Matem atica

Vers ao de 29 de janeiro de 2013.

Cap tulo 13

619/2069

ao: regra de Leibniz. E. 13.25 Exerc cio. Verique! Sugest Assim, se y e solu ca o de (13.164) segue que y (m) e solu ca o de (13.163). Conclu mos que as u nicas solu co es de (13.163) que s ao regulares em x = 0 s ao da forma
m) L( n (x) =

dm dm L ( x ) = n dxm dxm

ex

dn n x (x e ) dxn

(13.165)

au ltima igualdade sendo proveniente de (13.141) ou de (14.134). Os polin omios Ln s ao denominados polin omios de Laguerre associados. Os polin omios de Laguerre associados surgem, como dissemos, na resolu ca o da equa ca o de Schr odinger para o atomo de hidrog enio em coordenadas esf ericas. Vide Se ca o 19.8, p agina 909. Junto com as harm onicas esf ericas, denidas na Se ca o 14.2.2.1, p agina 652, os polin omios de Laguerre associados denem a forma dos orbitais eletr onicos do atomo de hidrog enio e (de forma aproximada) de atomos hidrogen oides. A forma desses orbitais e de import ancia fundamental no estudo de atomos e mol eculas e suas liga co es qu micas. Usando (13.140), e f acil constatar que
nm m m) L( n (x) = (1) (m )

k=0

(1)k

n! n xk . k! m + k

Mais propriedades dos polin omios de Laguerre associados ser ao estudadas na Se ca o 14.2.6, p agina 669.

JCABarata. Curso de F sica-Matem atica

Vers ao de 29 de janeiro de 2013.

Cap tulo 13

620/2069

13.4

Exerc cios Adicionais

2 E. 13.26 Exerc cio. Considere as equa co es diferenciais u (x) au(x) = 0 e u (x) + 0 u(x) = 0, com a C, 0 C, constantes e x C. Usando o m etodo de expans ao em s erie mostre que suas solu co es gerais s ao, respectivamente, u(x) = Aeax e u(x) = A cos(0 x) + B sen (0 x), onde A e B s ao constantes.

ao binomial E. 13.27 Exerc cio. Seja a bem conhecida expans

(1 + x) =
k=0

( + 1 k )k k x , k!

(13.166)

v alida para x C com |x| < 1 e para todo C, onde, para x C e n N0 , (x)n s ao os s mbolos de Pochhammer denidos em (13.147), p agina 612. Demonstre (13.166) resolvendo a equa c ao diferencial (1 + x)y y = 0 com a condi c ao y (0) = 1. Sugest ao. Verique que (1 + x) e solu c ao da equa c ao diferencial acima e satisfaz y (0) = 1. Depois resolva a mesma equa c ao, procurando solu co es na forma de uma s erie de pot encias na regi ao |x| < 1.

Mostre que quando = n N0 , um inteiro n ao-negativo, a solu c ao reduz-se a um polin omio, a saber, aquele denido pelo bin omio de Newton: n n (1 + x)n = xk . k
k=0

etodo de expans ao em s erie de pot encias mostre que a solu c ao da equa c ao diferencial E. 13.28 Exerc cio. Usando o m y (z ) + zy (z ) = 0 e y (z ) = c exp(z 2 /2), onde c e uma constante. E. 13.29 Exerc cio. Encontre, utilizando o m etodo de expans ao em s erie, a solu c ao geral da seguinte equa c ao diferencial u (x) ex u (x) + sin(x)u(x) = 0 . Em que regi ao a s erie de pot encias obtida para u(x) deve ser convergente? Justique. c ao u(x) = arcsen(x) E. 13.30 Exerc cio. Mostre que a fun
2
2

e a solu c ao da equa c ao diferencial

(1 x2 )u (x) xu (x) = 2 , com as condi co es iniciais u(0) = u (0) = 0. Usando o m etodo de expans ao em s erie para resolver a equa c ao, obtenha a expans ao de arcsen(x)
2

em uma s erie de pot encias


k=0

ck xk . Essa s erie coincide com a s erie de Taylor de arcsen(x)

em x = 0. Esse m etodo de determinar a expans ao em s erie de Taylor dessa fun c ao e muito mais simples que o m etodo direto, 2 envolvendo o c omputo das derivadas da fun c ao arcsen(x) em x = 0, e foi descoberto por Euler. Segundo [104], a s erie obtida j a era conhecida do matem atico Kowa Seki (16421708), contempor aneo de Newton. etodo de Frobenius determine a solu c ao geral da seguinte equa c ao diferencial: E. 13.31 Exerc cio. a) Pelo m x2 u (x) (1 + x)u(x) = 0 , b) Qual o raio de converg encia das s eries encontradas? Justique. c) Determine a solu c ao da mesma equa c ao que satisfaz a condi c ao u(0) = 0. H a solu co es para a condi c ao inicial u(0) = 1? Justique.

JCABarata. Curso de F sica-Matem atica

Vers ao de 29 de janeiro de 2013.

Cap tulo 13

621/2069

Ap endices
13.A Prova da Proposi c ao 13.1. Justicando os Polin omios de Legendre

Provaremos a Proposi ca o 13.1 apenas para o caso da s erie mutatis mutandis, id entica.
k=0

c2k z 2k , pois a demonstra ca o para a s erie


k=0

c2k+1 z 2k+1 e,

Caso R seja um inteiro n ao-negativo par, a s erie em (13.12) torna-se um polin omio e e, conseq uentemente, nita para todo z C.

Consideremos, ent ao, que R n ao e um inteiro n ao-negativo par. Tomemos a s erie em (13.12) somada, para simplicar, a partir de k = 2 e calculada em z = 1 (tomamos c0 = 1, sem perda de generalidade):

k=2

c2k = ( + 1)
N N

k=2

1 2k

k 1

l=1

( + 1) 2l(2l + 1)

Consideremos, para N > 2, c2k =


k=2 k=2 k 1 l=1

1 2k

k 1 l=1

( + 1) 2l(2l + 1)

Se ( + 1) 0 teremos que

( + 1) 2l(2l + 1)

1,

pois os fatores s ao positivos e maiores que 1. Logo,


N N

c2k =
k=2 N k=2

1 2k

k 1

l=1

( + 1) 1 2l(2l + 1)
N

k=2

1 . 2k

Portanto, como lim

k=2

1 diverge, isso prova que lim N 2k

c2k diverge, completando a prova.


k=2

claro que existe k0 N, k0 > 2, tal que Se ( + 1) > 0 devemos proceder de outra forma. E 0 < o que implica 1
(+1) 2l(2l+1) N

( + 1) < 1, 2k0 (2k0 + 1)

(13.A.1)

> 0 para todo l > k0 . Escolhendo N > k0 , podemos escrever


k0 N

c2k
k=2

=
k=2 k0

c2k +
k=k0 +1 k0 1

c2k

=
k=2

c2k +
l=1

( + 1) 2l(2l + 1)

k=k0 +1

1 2k

k 1

l=k0

( + 1) 2l(2l + 1)

(13.A.2)

Podemos escrever

k 1

l=k0

( + 1) 1 2l(2l + 1)

k 1

= exp
l=k0

ln 1

( + 1) 2l(2l + 1)

pois 1

(+1) 2l(2l+1)

> 0 para todo l k0 .

JCABarata. Curso de F sica-Matem atica

Vers ao de 29 de janeiro de 2013.

Cap tulo 13

622/2069

Agora, se 0 x M para algum 0 < M < 1, ent ao vale ln(1 x) x ln(1 M ) . M (13.A.3)

Isso pode ser provado de diversas formas, por exemplo usando a concavidade da fun ca o logaritmo (vide Cap tulo 5, p agina 233), que garante que ln a + (1 )b Com isso, e como 0 <
(+1) 2l(2l+1)

ln(a) + (1 ) ln(b) ,

para todo 0 1 e todo 0 < a < b. Tomando a = 1 M , b = 1 e = x/M , estabelece-se (13.A.3).


(+1) 2k0 (2k0 +1)

=: M , para todo l k0 , temos que exp ln(1 M ) M


k 1

k 1

exp
l=k0

ln 1

( + 1) 2l(2l + 1)

l=k0

( + 1) 2l(2l + 1)

Agora,
k 1 l=k0

( + 1) 2l(2l + 1)

l=k0

( + 1) < , 2l(2l + 1)

pois a s erie acima e convergente. Assim, denindo K :=


l=k0 k 1

( + 1) , teremos que 2l(2l + 1)


k 1

exp
l=k0

ln 1

( + 1) 2l(2l + 1)

exp

ln(1 M ) M

l=k0

( + 1) 2l(2l + 1)

exp

ln(1 M ) K M

j a que, por (13.A.1), ln(1 M ) < 0.

Dessa forma, retornando a (13.A.2), temos que


N k0 k0 1 N k 1

k=2

c2k

c2k
k=2

=
l=1 k0 1

( + 1) 2l(2l + 1)

k=k0 +1

1 exp 2k

l=k0

ln 1
N

( + 1) 2l(2l + 1) 1 . 2k

l=1

( + 1) 1 2l(2l + 1)

exp
N

ln(1 M ) K M

k=k0 +1

Como o limite lim

k=k0 +1

1 diverge, conclu mos que lim N 2k

c2k tamb em diverge, completando a prova.


k=2

13.B

Polin omios de Legendre: Provando (13.14)


m/2 (0) Pm (z ) = c0 ym (z ) = c0 k=0

Vamos considerar apenas o caso em que m e par, pois o caso em que m e mpar pode ser tratado de forma totalmente an aloga. Temos que z 2k (2k )!
k 1 l=0

2l(2l + 1) m(m + 1) ,

Como dissemos, a conven ca o e escolher c0 de modo que o coeciente do mon omio de maior grau do polin omio acima seja (2m)! . Assim, devemos ter m 2 2 (m!) 1 c0 m!
m 2 1

l=0

2l(2l + 1) m(m + 1)

(2m)! , 2m (m!)2

JCABarata. Curso de F sica-Matem atica

Vers ao de 29 de janeiro de 2013.

Cap tulo 13

623/2069

ou seja, c0 Com isso Pm (z ) =


k=0

(2m)! = m 2 m!
m/2

m 2 1

l=0

2l(2l + 1) m(m + 1)
m 2 1

z 2k (2m)! (2k )! 2m m!
m 2

l= k

2l(2l + 1) m(m + 1)

Fa camos agora a mudan ca de vari avel k


m/2

k . Ficamos com
m 2 1

Pm (z ) =
k=0

z m2k (2m)! (m 2k )! 2m m!
m 2

l= m 2 k

2l(2l + 1) m(m + 1)

Fa camos ainda a mudan ca de vari avel l


m/2

l. Obtemos,
k l=1 1

Pm (z ) =
k=0

z m2k (2m)! (m 2k )! 2m m!

(m 2l)(m 2l + 1) m(m + 1)

Entretanto, (m 2l)(m 2l + 1) m(m + 1) = 2l(2m 2l + 1) , como facilmente se v e. Agora, com isso,


k l=1 1 k 1

(m 2l)(m 2l + 1) m(m + 1)

=
l=1

2l(2m 2l + 1)
k

(1)k

l=1

1 2l

l=1

1 2m 2l + 1

(1)k (2k )!!

l=k+1 m

(2m 2l + 1)

l=1

(2m 2l + 1)
m l=k+1 mk l=1

(1)k (2k )!! (2m 1)!! (1)k (2k )!! (2m 1)!!

(2m 2l + 1)

ll+k

(2(m k ) 2l + 1)

= Assim,
m/2

(1)k (2(m k ) 1)!! . (2k )!! (2m 1)!! (2m)! (2(m k ) 1)!! m! (2k )!! (2m 1)!!

Pm (z ) =
k=0

(1)k z m2k 2m (m 2k )!

JCABarata. Curso de F sica-Matem atica

Vers ao de 29 de janeiro de 2013.

Cap tulo 13

624/2069

Vale, por em, (2m)! (2(m k ) 1)!! m! (2k )!! (2m 1)!! = (2m)! (2(m k ) 1)!! m! (2k )!! (2m 1)!! (2(m k ))!! (2(m k ))!!

(2m)! (2(m k ))! m! (2m 1)!! (2k )!! (2(m k ))!! (2m)!! (2m 2k )! m! (2k )!! (2(m k ))!! 2m m! (2m 2k )! m! 2k k ! 2mk (m k )! (2m 2k )! , k ! (m k )!

onde, na pen ultima passagem, usamos que (2p)!! = 2p p! para todo p N0 . Com isso,
m/2

Pm (z ) =
k=0

(1)k z m2k (2m 2k )! , 2m (m 2k )! k ! (m k )!

que e a express ao (13.14) para m par. O caso em que m e mpar e an alogo e e deixado como exerc cio.

13.C

Justicando os Polin omios de Hermite


(0) y (x) := 1

Tomaremos aqui z = x R e consideraremos apenas a s erie 2 x 2

k=2

x2k (2k )!

k 1

l=1

(4l ) ,

com R mas = 2m para m um inteiro positivo par (o que faz da s erie acima uma s erie innita), pois o tratamento (1) da s erie y e id entico. Seja s > 1, arbitr ario mas xo, e escolhamos k0 > 2 tal que 1 todo k0 > 2 enquanto que, se > 0, devemos tomar k0 > max Escrevemos
(0) y (x) := 1 4k0

> 1 e v alido para s . Note que se 0, isso (13.C.4)

s , 2 4(s 1)
k 1

2 x 2

k0

k=2

x2k (2k )!

l=1

(4l )

k=k0 +1

x2k (2k )!

k 1

l=1

(4l ) .

f E acil vericar que

k=k0 +1

x2k (2k )!

k 1

l=1

(4l ) =

4k1 x2k
k=k0 +1 k0 1

(k 1)! (2k )!

k 1

l=1

4l (k 1)! (2k )!
k 1

1 4

l=1

4l

4k x2k
k=k0 +1

l=k0

4l

JCABarata. Curso de F sica-Matem atica

Vers ao de 29 de janeiro de 2013.

Cap tulo 13

625/2069

Vamos agora nos concentrar na s erie


k=k0 +1

4k x2k

(k 1)! (2k )! 4l

k 1 l=k0

1 4k0

. Pela escolha de k0 , sabemos que para l k0 , 4l > 1 s

vale 1 e, portanto,

1 4l

k 1 l=k0

>

1 . skk0

Al em disso, (2k )! = (2k )!! (2k 1)!! = 2k k ! (2k 1)!! < 22k (k !)2 , pois (2k 1)!! = (2k 1)(2k 3)(2k 5) 1 = 2k k Logo,

1 2

3 2

5 2

1 < 2k k (k 1)(k 2) 1 . 2

4k x2k
k=k0 +1

(k 1)! (2k )!

k 1 l=k0

4l

>

sk0
k=k0 +1

1 k (k !)

x2 s

>

sk0
k=k0 +1

1 (k + 1)!

x2 s

sk0

s x2

k=k0 +1

1 (k + 1)!
k=k0 +1

x2 s 1 k!

k+1

sk0 +1 x2

x2 /s

k=0

x2 s

Tudo isso mostra que

(0) y (x)

e maior que

Kex

/s

x2

p(x)

, onde K e uma constante (que depende de , s e k0 ) e


(0)
2

p(x) e um polin omio de grau 2k0 + 2 em x. Como s e arbitr ario, vemos que o produto y ex 29 j a que podemos escolher 1/s > 1/2, tomando 1 < s < 2.

/2

diverge para |x| ,

No contexto do problema do oscilador harm onico na Mec anica Qu antica (vide Se ca o 19.7, p agina 907) esse compor(0) x2 /2 tamento e inaceit avel, pois o produto y e representa uma fun ca o de onda, que deve ser de quadrado integr avel em (0) R. Isso for ca-nos a tomar = 2m com m um inteiro positivo e par, de modo a reduzir y (x) a um polin omio. Para y (x) as considera co es s ao an alogas e n ao iremos repeti-las aqui.
(1)

13.D

Polin omios de Hermite: Provando (13.20)

Consideraremos apenas o caso em que m e par, pois o caso em que m e mpar e tratado analogamente. Para m par, tem-se m 2 2k k1 z (4l 2m) . Hm (z ) = (2)m/2 (m 1)!! 1 m z 2 2m (2k )!
k=2 l=1
29 Por

(13.C.4), tomar s pr oximo de 1 aumenta o grau do polin omio p(x), mas n ao altera o fato que y (x)ex

(0)

/2

diverge para |x|

JCABarata. Curso de F sica-Matem atica

Vers ao de 29 de janeiro de 2013.

Cap tulo 13

626/2069

Fazendo a mudan ca de vari aveis k

m 2

Tem-se que

Hm (z ) = (2)m/2 (m 1)!! 1 m z 2 2m
m 2 k 1 m 2 k 1

k , teremos

m 2 2

k=0

z m2k (m 2k )!

m 2 k 1

l=1

(4l 2m) .

l=1

(4l 2m)

(2)

m 2 k 1

l=1

(m 2l)

m 2 1

(2)

m 2 k 1

l=1 m 2 1

(m 2l) (m 2l )

l = m 2 k
m 2 1

l m 2 l

(2)

m 2 k 1

l=1

(m 2l)
k

2l
l =1

= (2) 2 k1
m

(m 2)!! . (2k )!!

Logo, Hm (z ) = (2)m/2 (m 1)!! 1 m z 2 2m


m 2 2

m 2 2

k=0

m ( m 2)!! z m2k (2) 2 k1 (m 2k )! (2k )!!

(2)
m 2

m 2

(m 1)!! 1 m z

+
k=0

(1)k m! (2z )m2k (m 2k )! k ! (13.D.5)

=
k=0

(1)k m! (2z )m2k , (m 2k )! k !

j a que m (m 1)!! (m 2)!! = m!, que (2k )!! = 2k k ! e que (2p)! (2p)!! (2p 1)!! = = 2p (2p 1)!! . p! p!

A express ao (13.D.5) coincide com (13.20) para m par. O caso em que m e mpar e an alogo e e deixado como exerc cio.

13.E

Porque deve ser um Inteiro Positivo na Equa c ao de Laguerre

Justicaremos aqui por que consideramos um inteiro positivo na equa ca o de Laguerre. Temos dois casos a tratar: a. < 0 e b. > 0 mas n ao-inteiro. Em aplica co es, especialmente na Mec anica Qu antica, a vari avel z e um n umero real positivo (uma coordenada radial). Vamos ent ao doravante tomar z real e positivo e escrever z = r > 0. Se n ao for um inteiro positivo a s erie (13.139) acima e uma s erie innita. Podemos escrever
n1 n1 l=1 n1 l=1

(1)n

l=0

( l) =

(l ) = (n 1)!

(13.E.6)

JCABarata. Curso de F sica-Matem atica

Vers ao de 29 de janeiro de 2013.

Cap tulo 13

627/2069

Se < 0, a u ltima express ao ca


n1

||(n 1)! e y1 (r) = 1 + || Agora,


1 n n=1

1+
l=1 n1

|| l || l rn .

1 n(n!)

1+
l=1

>

1 n+1

e 1+

|| l

> 1. Assim, y1 (r) > 1 + || 1 || r rn = 1 + (e 1 r) . ( n + 1)! r n=1

Disso conclu mos que y1 (r) cresce da ordem de er quando r . O problema com isso e que em v arias aplica co es tal comportamento e indesejado. No problema do atomo de hidrog enio da Mec anica Qu antica, por exemplo, o produto er/2 y1 (r) representa a fun ca o de onda radial de um el etron de momento angular nulo sob um potencial coulombiano30 . Pelo visto acima, se < 0 a fun ca o de onda cresceria para r pelo menos como e+r/2 , n ao podendo, assim, ser uma fun ca o de quadrado integr avel em R3 , uma condi ca o fundamental ligada ` a interpreta ca o probabil stica da Mec anica Qu antica. Assim, solu co es com < 0 devem ser descartadas nesse contexto. Tratemos agora do caso em que e positivo, mas n ao e um n umero inteiro. Por (13.E.6), podemos escrever, para n 1 2,
n1 21

(1)n

l=0

( l) = (n 1)!

l=1

n1

l=2

onde e o menor inteiro maior ou igual a . Assim,


2

y1 (r) = 1 +
n=1

(1)n (n!)2

n1 l=0

( l)

rn + L
n=2+1

com L :=

1 n (n!) .

n1

l=2

n 1 r , l

21

l=1

n1

A raz ao de escrevermos essa express ao dessa forma reside no fato que, agora,
l=2

e um produto de termos

positivos, sendo que, para l 2 tem-se 1 onde := 1


30 Vide

2 + ( ) 1 = = > = . 2 2 2 2 2

Se ca o 19.8, p agina 909, ou qualquer bom livro de Mec anica Qu antica.

JCABarata. Curso de F sica-Matem atica

Vers ao de 29 de janeiro de 2013.

Cap tulo 13

628/2069

Com isso, para a u ltima soma do lado direito vale n1 1 rn 1 n (n!) l


n=2+1 l=2

n=2+1

1 n2 n () r n (n!) 1 (r)n n (n!) 1 (r)n (n + 1)!

K
n=2+1

>

K
n=2+1

=
2+1

K r e P (r) r

onde K :=

, P (r) :=
n=0

1 (r)n e um polin omio de grau 2 + 1 e > 1/2. n!

Disso conclu mos que para r , |y1 (r)| cresce mais r apido que er com > 1/2. Assim, um produto como r/2 e y1 (r), que como dissemos representa a fun ca o de onda radial de um el etron de momento angular nulo sob um potencial coulombiano, n ao e de quadrado integr avel no espa co R3 , uma condi ca o fundamental ligada ` a interpreta ca o probabil stica da Mec anica Qu antica. Assim, solu co es com > 0, mas n ao-inteiro, devem tamb em ser descartadas nesse contexto.

13.F

Polin omios de Tchebychev: Obtendo (13.39) a Partir de (13.36)(13.38)


k 1 l=0

Trataremos apenas o caso em que m e par e m 2, o caso mpar sendo an alogo. O fator em (13.36) pode ser reescrito da seguinte forma:
k 1 k 1 k 1 k 1 k 1

(2l)2 m2 que ocorre

l=0

(2l)2 m2

=
l=0

2l + m
l=0

2l m

= (1)k

2l + m
l=0 l=0

m 2l m!! (2k 2 + m)!! . (m 1)!! (m 2k )!!

= (1)k E. 13.32 Exerc cio. Justique! Com isso e com a mudan ca de vari avel k = m/2 j , escrevemos
m/2 (0) ym (z ) = j =0

z m2j (1)m/2j

(2m 2j 2)!! m!! . (m 2j )! (2j )!! (m 1)!!


(0)

1)!! para que o coeciente do termo de maior grau torne-se 2m1 , obtemos por (1)m/2 m (m m!! m/2

m!! O coeciente do termo de maior grau (que corresponde a j = 0) e (1)m/2 2m1 m (m 1)!! . Assim, multiplicando-se ym (z )

Tm (z ) =
j =0

z m2j (1)j m

(2m 2j 2)!! . (m 2j )! (2j )!!

JCABarata. Curso de F sica-Matem atica

Vers ao de 29 de janeiro de 2013.

Cap tulo 13

629/2069

Usando agora o fato que (2a)!! = 2a a!, a N, obtemos nalmente


m/2

Tm (z ) =
j =0

(2z )m2j

m (m j 1)! (1)j , 2 (m 2j )! j !

que e (13.39).

Você também pode gostar